File:  [Local Repository] / db / baza / rusvuz17.txt
Revision 1.3: download - view: text, annotated - select for diffs - revision graph
Tue Nov 14 13:36:07 2017 UTC (6 years, 6 months ago) by rubashkin
Branches: MAIN
CVS tags: HEAD
*** empty log message ***

Чемпионат:
Открытый чемпионат вузов России по ЧГК. 2016/17

Дата:
00-000-2016

Тур:
Первый игровой день. 1 тур

Дата:
30-Sep-2016

Редактор:
Илья Иванов (Пермь)

Инфо:
Редактор благодарит за помощь в подготовке пакета Виктора Абдураманова
(Березники), Арсэна Атнагулова (Уфа), Олега Белослудцева, Константина
Костенко, Вадима Опутина, Александра Пономарёва, Александра Шишкина,
Юлию Шучалову и Дениса Яруллина (все - Пермь), Александра Быкова и Ирину
Пинегину (оба - Киров), Андрея Ефремова (Могилев), Андрея Кокуленко
(Омск), Александра Мудрого (Черновцы), Константина Науменко (Киев),
Ксению Рудич (Москва), Константина Сахарова (Ивантеевка), Ульяну
Фабричнину (Санкт-Петербург), а также Дмитрия Башука и команду "От винта
- Братья по фазе" (Харьков).

Вопрос 1:
В пражском ресторане "Vytopna" [вЫтопна] процесс доставки пива
механизирован: клиенты получают свое пиво, когда происходит ЭТО. Кто
создал "ЭТО" в конце XIX века?

Ответ:
[Братья] Люмьер.

Комментарий:
За доставку пива в ресторане отвечает миниатюрная железная дорога -
поезд с бокалом подъезжает к посетителю, и тот может забрать стакан.
"Прибытие поезда" - знаменитый фильм братьев Люмьер.

Источник:
   1. http://luxlux.net/prazhskiy-restoran-vytopna-poezd-vmesto-ofitsianta-35437/
   2. http://ru.wikipedia.org/wiki/Прибытие_поезда_на_вокзал_Ла-Сьота

Автор:
Игорь Демьянцев (Гомель)

Вопрос 2:
В американском комедийном мультсериале сотрудники одной организации
используют фургон, предлагающий свежие буррито. Назовите эту
организацию.

Ответ:
ФБР.

Зачет:
FBI; Федеральное Бюро Расследований; Federal Bureau of Investigation.

Комментарий:
Спецслужбы часто используют фургоны для слежки. В мультсериале агенты
ФБР (по-английски - FBI) умудрились использовать фургон с надписью
"Fresh Burritos Instantly" [фреш буррИтос Инстантли] ("Свежие буррито
немедленно"), сокращенно - FBI.

Источник:
Мультсериал "Симпсоны", s25e01.

Автор:
Илья Иванов (Пермь)

Вопрос 3:
В 1894 году астроном Эндрю Дуглас предложил построить обсерваторию в
городке Флагстафф в Аризоне. Рассказывают, что в конце жизни Дуглас
признался, что во Флагстаффе его интересовали только луны. В одном из
слов предыдущего предложения мы пропустили две буквы. Напишите это слово
в исходном варианте.

Ответ:
Салуны.

Комментарий:
Территория славилась своими салунами - Аризона была частью дикого
Запада, а дело было в конце XIX века. Вот такой вот "научный" критерий
выбора для обсерватории.

Источник:
   1. http://ru.wikipedia.org/wiki/Обсерватория_Лоуэлла
   2. http://www.peoples.ru/science/astronomy/andrew_ellicott_douglas/

Автор:
Илья Иванов (Пермь)

Вопрос 4:
По несколько парадоксальному утверждению комментатора, эксцентричный
Фабьен Бартез иногда устраивал в воротах ЕЕ. Назовите ЕЕ словом с
удвоенной согласной.

Ответ:
Буффонада.

Комментарий:
Бартез в воротах постоянно "чудил". Джанлуиджи Буффон - другой
легендарный футбольный голкипер.

Источник:
Трансляция финала чемпионата мира по футболу 1998 года Франция -
Бразилия на телеканале "Матч-ТВ", 08.07.2016 г.

Автор:
Илья Иванов (Пермь)

Вопрос 5:
[Ведущему: обязательно произнести название игры полностью!]
   В начале игры "Шерлок Холмс: Преступления и наказания" главный герой,
стреляя по вазам, приветствует инспектора Лестрейда, чем изрядно
удивляет последнего. Какие три слова мы пропустили в этом вопросе?

Ответ:
С завязанными глазами.

Зачет:
С закрытыми глазами; другие синонимичные ответы из трех слов.

Комментарий:
Несмотря на завязанные глаза, Холмс определяет, что в комнату вошел
именно инспектор Лестрейд, - главным образом, благодаря бряцанью
наручников, звуку шагов и тому, что "скрипучая" ступенька не скрипнула
(значит, гость бывает в доме часто).

Источник:
Игра "Sherlock Holmes: Crimes and Punishments".

Автор:
Илья Иванов (Пермь)

Вопрос 6:
Герой фильма "Космос как предчувствие" знакомится в поезде с молодым
летчиком Юрием. Когда тот направляется к выходу, наблюдательный герой
окликает его и указывает на один из НИХ. Назовите ИХ.

Ответ:
Шнурки.

Комментарий:
Создатели фильма явно решили сделать отсылку к известной истории, когда
у Гагарина, шедшего по аэродрому Внуково к правительственной трибуне,
развязался шнурок. Впрочем, существует версия, что это были подтяжки для
носков - однако в фильме, конечно, фигурировали именно шнурки.

Источник:
   1. Х/ф "Космос как предчувствие" (2005), реж. Алексей Учитель.
   2. http://www.retroportal.ru/gagarin_14_04_1961.shtml

Автор:
Илья Иванов (Пермь)

Вопрос 7:
Герой французской комедии любезно отдает молодой немке солонку и
перечницу. Недовольная жена язвительно спрашивает, почему же он не
решился предложить девушке арию. В одном из слов предыдущего предложения
мы пропустили шесть букв. Напишите это слово в исходном варианте.

Ответ:
Лотарингию.

Комментарий:
Лотарингия - знаменитая провинция, которая (вместе с Эльзасом) долгое
время была причиной ожесточенных сражений и споров между Францией и
Германией.

Источник:
Х/ф "Каникулы маленького Николя" (2014), реж. Лоран Тирар.

Автор:
Илья Иванов (Пермь)

Вопрос 8:
В уже упомянутой нами игре на стене в комнате Шерлока Холмса можно
увидеть ИКС, написанный в 1872 году. Назовите ИКС двумя словами.

Ответ:
Портрет Достоевского.

Комментарий:
Игра называется "Шерлок Холмс: Преступления и наказания", что является
отсылкой к знаменитому роману Федора Михайловича.

Источник:
   1. Игра "Sherlock Holmes: Crimes and Punishments".
   2. https://www.wikiart.org/ru/vasiliy-perov/portret-pisatelya-fedora-mikhaylovicha-dostoevskogo-1872

Автор:
Илья Иванов (Пермь)

Вопрос 9:
На карикатуре Жана Эффеля Бог, прогуливающийся среди хаоса, замечает,
что мир - это ИКС. Одним из лучших ИКСОВ 2015 года был признан
"нипричёмыш". Какое слово греческого происхождения мы заменили ИКСОМ?

Ответ:
Неологизм.

Комментарий:
На карикатуре, посвященной сотворению мира, Бог достаточно справедливо
называет создаваемый им мир неологизмом. "Нипричёмыш" - скучный, тихий
человек, всегда держащийся подальше от любых волнений, - был признан
Экспертным советом при Центре творческого развития русского языка одним
из лучших неологизмов 2015 года.

Источник:
   1. Ж. Эффель. Сотворение мира. - М.: Изобразительное искусство, 1989.
   2. https://snob.ru/profile/27356/blog/101993

Автор:
Виктор Абдураманов (Березники)

Вопрос 10:
По анекдотической версии Александра Экстера, жена однажды увидела
картину мужа из цикла работ о женской бане и пришла в ярость. Какое
произведение, согласно этой версии, было создано в результате?

Ответ:
"Черный квадрат".

Комментарий:
И в ярости замазала полотно черной краской. Так Малевич и придумал свой
"Черный квадрат" (во что, конечно, верится с трудом).

Источник:
http://www.vokrugsveta.ru/article/205155/

Автор:
Илья Иванов (Пермь)

Вопрос 11:
Одна из шведских церквей недавно вывесила объявление с напоминанием, что
по всё еще действующему закону 1280 года никто не может быть взят в плен
в церкви. Отдельно подчеркнуто, что под защиту закона подпадают и ОНИ.
Назовите ИХ одним словом.

Ответ:
Покемоны.

Комментарий:
Судя по всему, любители поохотиться на покемонов делали это и в церкви.

Источник:
http://www.diary.ru/~Logovojuni/p209907950.htm

Автор:
Константин Костенко (Гремячинск - Пермь)

Вопрос 12:
Опера ДжАкомо Пуччини "ТОска" заканчивается прыжком главной героини с
высокой башни. По легенде, конец одной из постановок "ТОски" привел
публику в восторг. Причиной этого стало использование ЕГО. Назовите ЕГО
одним словом.

Ответ:
Батут.

Комментарий:
Исполнительница главной роли спрыгнула из декорационной башни, однако
вместо привычных матрасов кто-то додумался установить батут. В итоге
певица три или четыре раза подпрыгнула на нем, что не осталось
незамеченным публикой.

Источник:
Э. Ланди. Тайная жизнь великих композиторов.
http://www.flibusta.is/b/390922/read

Автор:
Илья Иванов (Пермь)

Тур:
Первый игровой день. 2 тур

Дата:
30-Sep-2016

Редактор:
Андрей Ефремов (Могилев)

Инфо:
Редактор благодарит за помощь в тестировании Валерия Семёнова, Сергея
Дубелевича и команду "Солянка" (все - Минск), команду "От винта - Братья
по фазе" (Харьков) и Лидию Иоффе (Хайфа).

Вопрос 1:
В научно-популярном фильме параллельная эволюция вирусов и человеческого
организма сравнивается с НЕЙ. Назовите ЕЕ двумя словами, начинающимися
на соседние буквы алфавита.

Ответ:
Гонка вооружений.

Комментарий:
Вирусы и человеческие клетки на протяжении тысячелетий вынуждены были
изобретать всё новые и новые механизмы атаки и защиты, обмана и его
распознавания.

Источник:
Эфир телеканала "Наука 2.0", сериал "Тайная вселенная", серия
"Путешествие внутрь клетки", эфир от 11.02.2015 г.

Автор:
Андрей Ефремов (Могилев)

Вопрос 2:
Автор исторического романа сравнил ЕЕ с заточённой в монастырь царицей,
которую ветреный и капризный властелин променял на холодную,
змеиноглазую разлучницу. Назовите ЕЕ одним словом.

Ответ:
Москва.

Комментарий:
Как известно, в XVIII веке Петр I перенес столицу в Санкт-Петербург.

Источник:
Б. Акунин. Любовница смерти. http://www.flibusta.is/b/500333/read

Автор:
Андрей Ефремов (Могилев)

Вопрос 3:
В статье об ИКСЕ упоминаются коронарные артерии. В произведении XIV века
обитатели ИКСА стараются прикрыть один бок другим. Назовите ИКС двумя
словами.

Ответ:
Третий круг.

Комментарий:
Третий круг кровообращения обслуживает само сердце. Во втором
предложении речь идет о третьем круге ада в описании Данте, в который
помещены чревоугодники, стыдящиеся своей полноты. Вопрос в туре идет
третьим - это дополнительная подсказка.

Источник:
   1. http://www.science-education.ru/34-1342
   2. https://otvet.mail.ru/question/47811228

Автор:
Андрей Ефремов (Могилев)

Вопрос 4:
На карикатуре, действие которой происходит весной, ПЕРВЫЕ испуганно
смотрят на появляющиеся из земли ВТОРЫЕ. Какие однокоренные слова мы
обозначили как "ПЕРВЫЕ" и "ВТОРЫЕ"?

Ответ:
Снеговики, подснежники.

Источник:
http://pikabu.ru/story/predvestniki_smerti_1944655

Автор:
Андрей Ефремов (Могилев)

Вопрос 5:
В одном из стихотворений Владимир Маяковский удивляется: неужели "зуб
революций ступился о НИХ"? Какая денежная единица получила свое название
от НЕЕ?

Ответ:
Крона.

Комментарий:
ОНА - это корона. Как известно, прокатившаяся по Европе волна революций
способствовала падению не одной монархии.

Источник:
   1. В.В. Маяковский. Радоваться рано.
http://www.feb-web.ru/feb/mayakovsky/texts/ms0/ms2/ms2-016-.htm
   2. http://ru.wikipedia.org/wiki/Крона_(денежная_единица)

Автор:
Андрей Ефремов (Могилев)

Вопрос 6:
В вопросе словом "ИКС" заменено другое слово.
   Однажды в компьютерной игре "Герои меча и магии - 5" автору вопроса
встретился ИКС дьяволят. Назовите ИКС, созданный в 1831 году королевским
приказом.

Ответ:
[Французский] Иностранный [легион].

Комментарий:
Словосочетание "легион дьяволят" отсылает к цитате из Евангелия "Имя мне
- легион". Иностранный легион был создан по приказу Луи-Филиппа I, а
Почетный легион - еще при Наполеоне I.

Источник:
   1. ЛОАВ (пользовательская карта "Живее всех живых").
   2. http://ru.wikipedia.org/wiki/Имя_мне_%E2%80%94_легион
   3. http://ru.wikipedia.org/wiki/Французский_Иностранный_легион

Автор:
Андрей Ефремов (Могилев)

Вопрос 7:
В сентябре 1957 года сразу после аварии на химкомбинате "Маяк" в
челябинских газетах появились сообщения о НЕМ. ОНО бывает двух видов,
которые на самом деле вовсе не симметричны. Назовите ЕГО двумя словами,
начинающимися на разные буквы.

Ответ:
Полярное сияние.

Комментарий:
Советские газеты пытались дезинформировать население, выдав техногенную
катастрофу за природное явление. До недавнего времени считалось, что
полярные сияния в северном и южном полушарии являются симметричными.
Однако одновременное наблюдение полярного сияния в мае 2001 года из
космоса со стороны северного и южного полюсов показало, что северное и
южное сияния существенно отличаются друг от друга.

Источник:
   1. http://ru.wikipedia.org/wiki/Кыштымская_авария
   2. http://ru.wikipedia.org/wiki/Полярное_сияние

Автор:
Андрей Ефремов (Могилев)

Вопрос 8:
Описывая шумную толпу в лондонском порту, Филип Депуа пишет, что
невидимая ОНА поднималась до самого неба. Назовите ЕЕ двумя словами.

Ответ:
Вавилонская башня.

Комментарий:
Писатель упоминает "разноязыкий гомон". Слово "толпа" в вопросе -
подсказка.

Источник:
Ф. Депуа. Тайна короля Якова. http://www.flibusta.is/b/254925/read

Автор:
Андрей Ефремов (Могилев)

Вопрос 9:
При создании ИХ в резиденции сёгуна гвозди располагали в виде
перевернутой буквы V [ви] для имитации щебета различных птиц, по
которому можно было достаточно точно определить местонахождение
посетителей. Назовите ИХ двумя словами, начинающимися на одну и ту же
букву.

Ответ:
Поющие полы.

Зачет:
Птичьи полы.

Комментарий:
Поющие (соловьиные) полы были созданы в замке сёгунов в Киото, в первую
очередь, для того чтобы враги не могли пробраться туда незамеченными. В
зависимости от того, голос какой именно птицы был слышен, можно было
точно определить местонахождение проникнувшего в замок.

Источник:
https://historyporn.dirty.ru/poiushchie-poly-na-strazhe-siogunov-540719/

Автор:
Андрей Ефремов (Могилев)

Вопрос 10:
Престарелый герой рассказа Стивена Кинга сравнивает свое тело с
разрушенным ИКСОМ. Самый высокий ИКС был создан в Майами. Назовите ИКС.

Ответ:
Замок из песка.

Зачет:
Синонимичные ответы.

Комментарий:
Известно выражение "песок сыплется". Город Майами известен своими
пляжами. Кстати, предыдущий вопрос тоже был о зАмке.

Источник:
   1. С. Кинг. Человек в черном костюме.
http://www.flibusta.is/b/182301/read
   2. http://lenta.ru/news/2015/10/27/zamok/

Автор:
Андрей Ефремов (Могилев)

Вопрос 11:
Блиц.
   На одном специализированном обучающем сайте есть цикл детских
стихотворений.
   1. В одном из стихотворений ОНА "словно подлодка, стреляет прямой
наводкой". Назовите ЕЕ одним словом.
   2. В одном из стихотворений ОН предпочитает рукопашную. Назовите ЕГО
одним словом.
   3. В одном из стихотворений ОНА решает, кем стать. Назовите ЕЕ одним
словом.

Ответ:
   1. Ладья.
   2. Король.
   3. Пешка.

Комментарий:
В шахматах ладья контролирует вертикаль и горизонталь, король бьет
только соседние клетки, а пешка, дойдя до последней горизонтали,
превращается в одну из четырех других фигур.

Источник:
   1. http://fenix64.com/stix-o-lade/
   2. http://fenix64.com/stix-pro-korolya-2/
   3. http://fenix64.com/stix-pro-peshku/

Автор:
Андрей Ефремов (Могилев)

Вопрос 12:
Решительно настроенный президент США Уильям Тафт считал, что "ЭТО" может
успокоить забастовку рабочих. Обычно за один сеанс ЭТОГО устанавливают
от четырех до двенадцати... Кого?

Ответ:
Пиявок.

Комментарий:
Речь идет о кровопускании. Тафт ратовал за применение силы в отношении
бастующих.

Источник:
   1. http://ru.wikipedia.org/wiki/Тафт,_Уильям_Говард
   2. http://ru.wikipedia.org/wiki/Кровопускание

Автор:
Андрей Ефремов (Могилев)

Тур:
Первый игровой день. 3 тур

Дата:
30-Sep-2016

Редактор:
Дмитрий Башук (Харьков)

Инфо:
Редактор благодарит за помощь в подготовке вопросов команду "От винта -
Братья по фазе" (Харьков).

Вопрос 1:
Шутники утверждают, что для грузчиков характерно раздельное питание.
Когда они разгружают картошку, то едят картошку, когда разгружают сыр -
едят сыр. А когда разгружают кирпич, то это - ОН. Назовите ЕГО двумя
словами.

Ответ:
Разгрузочный день.

Комментарий:
"Разгрузочный день", как и "раздельное питание", - устойчивое понятие в
диетологии.

Источник:
"Арт-Мозаика", 2016, N 34.

Автор:
Дмитрий Башук (Харьков)

Вопрос 2:
Одну из актрис Фаина Раневская причислила к отряду молеобразных и
объяснила это тем, что у той все мысли только об АЛЬФАХ. Стихотворение
Семена Кирсанова об АЛЬФЕ заканчивается словами "... чтоб не мог мороз
ущипнуть тебя". Назовите АЛЬФУ.

Ответ:
Шуба.

Источник:
   1. Ф.Г. Раневская. Мудрые остроты Раневской.
http://www.flibusta.is/b/380824/read
   2. С.И. Кирсанов. Нащот шубы.
http://www.flibusta.is/b/343461/read#t21

Автор:
Дмитрий Башук (Харьков)

Вопрос 3:
Для десятилетнего Нила Сети, исполнившего роль Маугли в недавнем фильме
"Книга джунглей", костюмеры изготовили двадцать вариантов ИХ, которые
меняли в зависимости от эпизода. Назовите ИХ двумя словами.

Ответ:
Набедренные повязки.

Комментарий:
Других костюмов Маугли и не надо. :-)

Источник:
"Телескоп", 2016, N 16.

Автор:
Дмитрий Башук (Харьков)

Вопрос 4:
У основания кафедры венского собора Святого Стефана есть скульптурное
изображение человека с циркулем в руке. Назовите это изображение словом,
происходящим от древнегреческого и французского слов.

Ответ:
Автопортрет.

Комментарий:
(pic: 20160953.jpg)
   Создатель кафедры каменотёс Антон Пильграм изобразил себя взирающим
на дело своих рук; слово "автопортрет" происходит от древнегреческого
"autos" и французского "portrait".

Источник:
   1. Е.Н. Грицак. Вена. http://www.flibusta.is/b/116707/read
   2. http://ru.wiktionary.org/wiki/автопортрет

Автор:
Дмитрий Башук (Харьков)

Вопрос 5:
[Ведущему: при чтении явно указать запятую после первого пропуска.]
   Опальный поэт из книги Артуро Переса-Реверте в одном из эпизодов так
каламбурит о себе: "Дела идут [пропуск], дела [пропуск] дышат...".
Заполните пропуски - двумя словами каждый.

Ответ:
"... на лад...", "... на ладан...".

Комментарий:
Такой вот грустный каламбур.

Источник:
А. Перес-Реверте. Капитан Алатристе.
http://www.flibusta.is/b/456025/read

Автор:
Дмитрий Башук (Харьков)

Вопрос 6:
Материал журнала "Вокруг света" под заголовком "Личное влияние" посвящен
ИМ. Кстати, одна из ИХ разновидностей называлась словом, однокоренным к
слову из приведенного заголовка. Назовите ИХ.

Ответ:
Маски.

Комментарий:
Металлическая маска под названием "личина" была частью военного шлема.

Источник:
   1. "Вокруг света", 2016, N 7.
   2. http://ru.wikipedia.org/wiki/Личина

Автор:
Дмитрий Башук (Харьков)

Вопрос 7:
Площадь ЕЕ составляет около 4152 квадратных километров и с каждым годом
увеличивается. В 1991 году румынская часть, составляющая примерно 83%
общей площади, была признана ЮНЕСКО объектом Всемирного природного
наследия. Назовите ЕЕ двумя словами, которые начинаются на одну и ту же
букву.

Ответ:
Дельта Дуная.

Комментарий:
Остальные 17% принадлежат Украине; с каждым годом береговая линия
Черного моря смещается метров на сорок вглубь моря из-за наносов.

Источник:
http://ru.wikipedia.org/wiki/Дельта_Дуная

Автор:
Александр Лисянский (Харьков)

Вопрос 8:
В отличие от математики, в жизни минус на минус не всегда дает плюс.
Видимо, поэтому, согласно закону города Лик-Спрингс в штате Индиана,
выпуская на улицу черных котов в определенный день, хозяева обязаны
надевать на своих питомцев колокольчики. Назовите этот день абсолютно
точно.

Ответ:
Пятница, 13-е.

Комментарий:
Авторы закона, видимо, считают, что если нельзя избежать негативного
влияния пятницы, 13-го, то хотя бы черных котов, собирающихся перейти
дорогу, горожане в этот день заблаговременно обнаружат по звуку
колокольчиков. :-)

Источник:
http://www.koshsps.ru/blackcat.php

Автор:
Дмитрий Башук (Харьков)

Вопрос 9:
Персонаж книги Хольма Ван Зайчика рассуждает: "Невозможно представить
себе в просторах Запретного города, скажем, велосипед... это вопиюще
несообразно и даже несколько оскорбительно - как если бы, скажем, ИКС
вдруг запел...". Чуть больше сорока пяти лет назад так и произошло.
Назовите ИКСА.

Ответ:
Иисус Христос.

Комментарий:
В 1970 году была создана рок-опера "Иисус Христос - суперзвезда".

Источник:
   1. Х. Ван Зайчик. Дело Судьи Ди. http://www.flibusta.is/b/73894/read
   2. http://ru.wikipedia.org/wiki/Иисус_Христос_%E2%80%94_суперзвезда

Автор:
Дмитрий Башук (Харьков)

Вопрос 10:
Одной из причин распада этой группы в апреле 1981 года Лоуренс ДжУбер
считает то, что ее лидер стал бояться концертов (цитата): "Ведь ему
приходилось бы каждые десять минут вздрагивать, ожидая, что какой-нибудь
придурок выстрелит в него из пистолета". Назовите эту группу.

Ответ:
"Wings".

Комментарий:
Джубер - гитарист этой группы. Убийство Джона Леннона, произошедшее 8
декабря 1980 года, сильно потрясло Пола Маккартни, создавшего "Wings" в
1971 году, вскоре после распада "Битлз".

Источник:
http://ru.wikipedia.org/wiki/Wings

Автор:
Дмитрий Башук (Харьков)

Вопрос 11:
На западе США колючую проволоку часто использовали в качестве АЛЬФЫ,
чтобы не беспокоиться при возможных порывах ветра. Назовите АЛЬФУ двумя
словами.

Ответ:
Бельевая веревка.

Комментарий:
Шипы колючей проволоки легко удерживали сохнущую на ветру одежду.

Источник:
http://warspot.ru/2560-shipy-bez-roz

Автор:
Максим Евланов (Харьков)

Вопрос 12:
"Ты и есть мой дом" - такова, по мнению персонажа Макса Фрая, идеальная
ОНА. Премьера "ЕЕ" состоялась 30 декабря 1984 года. Назовите ЕЕ.

Ответ:
Формула любви.

Комментарий:
Во втором случае речь идет о фильме Марка Захарова.

Источник:
   1. М. Фрай. Сказки старого Вильнюса II.
http://www.flibusta.is/b/431642/read
   2. http://ru.wikipedia.org/wiki/Формула_любви

Автор:
Дмитрий Башук (Харьков)

Тур:
Второй игровой день. 1 тур

Дата:
11-Nov-2016

Редактор:
Артем Матухно (Одесса)

Инфо:
Редактор благодарит за помощь и дельные советы во время тестирования:
Дениса Гончара, Андрея Дунаева, Ирину Буртненко, Игоря Гайдута, Сергея
Гайкова, Елену Григоращенко, Андрея Герасименко, Евгения Криворученко,
Алексея Топышева, Андрея Ландера, Влада Гаврилова.

Вопрос 1:
Для одной традиции компания "Гиннесс" использует вместо ЭТОГО продукцию
собственного производства. В США в годы "сухого закона" вместо ЭТОГО
использовали кока-колу. Назовите ЭТО.

Ответ:
Шампанское.

Зачет:
Бутылка шампанского.

Комментарий:
Речь идет о традиции "крещения корабля" с помощью бутылки шампанского.
Пивоваренная фирма "Гиннесс" использует при крещении своих судов,
предназначенных для перевозки пива, собственное пиво. В добрый путь,
друзья!

Источник:
http://www.rg-rb.de/index.php?option=com_rg&task=item&id=15792&Itemid=0

Автор:
Артем Матухно (Одесса)

Вопрос 2:
В 1934 году подготовленная тренером КатрИн Кёртис группа из шестидесяти
человек под названием "Современные русалки" с успехом выступила на
Всемирной выставке в Чикаго. Какое словосочетание из двух слов впервые в
истории использовал диктор НОрман Росс, комментируя это выступление?

Ответ:
Синхронное плавание.

Комментарий:
Правда, сама Катрин предпочитала термин "ритмическое плавание".

Источник:
David Goldblatt. How to Watch the Olympics: An Instant Initiation into
Every Sport at Rio-2016.
https://books.google.ru/books?id=RqaCCwAAQBAJ&pg=PT366#v=onepage&q&f=false

Автор:
Артем Матухно (Одесса)

Вопрос 3:
[Ведущему: начинать читать текст вопроса со слова "внимание" - это не
ошибка и не опечатка.]
   Внимание, вопрос номер два!
   Рассказывают, что некоторое время ОН, уроженец Екатеринославской
губернии, специально посещал школьные уроки, где, сидя с блокнотом на
задних партах, что-то зарисовывал. Назовите ЕГО.

Ответ:
[Федор Павлович] Решетников.

Комментарий:
Художник посещал уроки в школе перед написанием своей известной картины
"Опять двойка". Ведущий намеренно ошибся, чуть не сделав третий вопрос
"опять вторым".

Источник:
http://www.vokrugsveta.ru/article/252015/

Автор:
Артем Матухно (Одесса)

Вопрос 4:
Военачальника Николая Муравьёва-Карсского и декабриста Петра Колошина
объединяла настолько близкая дружба, что даже своих коней они назвали в
честь ИКСОВ. В честь ИКСОВ названы две соседние вершины-четырехтысячники
в Альпах. Назовите имена ИКСОВ.

Ответ:
Кастор, Поллукс.

Зачет:
Кастор, Полидевк.

Комментарий:
ИКСЫ - Диоскуры. Муравьёв-Карсский и Колошин были настолько близкими
друзьями, что часто называли друг друга братьями. Мифические герои,
близнецы Кастор и Поллукс, были известны своей неразлучной дружбой.

Источник:
   1. Н.Н. Муравьёв-Карсский. Собственные записки. 1811-1816.
http://www.flibusta.is/b/450099/read
   2. http://en.wikipedia.org/wiki/Castor_(mountain)
   3. http://en.wikipedia.org/wiki/Pollux_(mountain)

Автор:
Артем Матухно (Одесса)

Вопрос 5:
По словам Юрия Рылёва, автора книги "6000 изобретений XX и XXI веков,
изменивших мир", разработка противозачаточных таблеток в середине
прошлого века стала залпом "Авроры" для НЕЕ. Назовите ЕЕ точно.

Ответ:
Сексуальная революция.

Источник:
Ю.И. Рылёв. 6000 изобретений XX и XXI веков, изменившие мир.
http://www.flibusta.is/b/377039/read

Автор:
Артем Матухно (Одесса)

Вопрос 6:
Название одного магазина, предоставляющего услуги по ремонту смартфонов,
представляет собой английское слово-неологизм. Какой фрукт присутствует
на логотипе этого магазина?

Ответ:
Груша.

Зачет:
Pear.

Комментарий:
Магазин называется "re:Pear" [ре пЭа], что созвучно английскому глаголу
"repair" [репЭа] - чинить, ремонтировать.

Источник:
Личные наблюдения автора вопроса в Стокгольме.

Автор:
Артем Матухно (Одесса)

Вопрос 7:
Назовите многократного олимпийского чемпиона, чья фамилия, по одной из
версий, могла быть образована от прозвища, которым когда-то называли
слишком прилипчивого зануду.

Ответ:
[Евгений Викторович] Плющенко.

Комментарий:
Прилипчивый, словно плющ. Евгений Плющенко - многократный олимпийский
чемпион в фигурном катании.

Источник:
   1. Т.Ф. Ведина. Энциклопедия русских фамилий. Тайны происхождения и
значения. http://www.flibusta.is/b/154189/read
   2. http://ru.wikipedia.org/wiki/Плющенко,_Евгений_Викторович

Автор:
Артем Матухно (Одесса)

Вопрос 8:
[Ведущему: при чтении выделить интонацией словосочетание "не он".]
   Неизвестно, нравилось ли это произведение издателю, но в изначальном
авторском варианте был не ОН, а "ключ". В каком южном городе ОН
находится до сих пор?

Ответ:
Бахчисарай.

Комментарий:
Речь идет о Бахчисарайском фонтане. Пушкин изначально назвал свою поэму
"Бахчисарайский ключ", а в "фонтан" она превратилась по воле издателя -
князя Вяземского. Возможно, он считал, что авторский вариант был, как
говорится, "не фонтан".

Источник:
http://www.vokrugsveta.ru/vs/article/8482/

Автор:
Артем Матухно (Одесса)

Вопрос 9:
Согласно "Словарю морского жаргона", АЛЬФА - это прозвище
"привилегированной женской особы, работающей в команде судна". АЛЬФОЙ
была персонаж серии детских книг, написанных в середине прошлого века.
Назовите АЛЬФУ двумя словами.

Ответ:
Капитанская дочка.

Комментарий:
АЛЬФА - капитанская дочка. Пеппи была дочкой капитана Эфраима
Длинныйчулок.

Источник:
   1. Н.А. Каланов. Словарь морского жаргона. - М.: Моркнига, 2010.
   2. http://ru.wikipedia.org/wiki/Пеппи_Длинныйчулок

Автор:
Артем Матухно (Одесса)

Вопрос 10:
В одном кроссворде слово "рикша" определяется как "таксист, которого
[ДВА СЛОВА ПРОПУЩЕНО]". А кого [ДВА СЛОВА ПРОПУЩЕНО], согласно русской
пословице?

Ответ:
Волка.

Комментарий:
Пропущены слова "ноги кормят". Пословица - "Волка ноги кормят".

Источник:
   1. http://www.kotvet.ru/odn/145
   2. http://slovarick.ru/205/

Автор:
Артем Матухно (Одесса)

Вопрос 11:
По одной из версий, ИКС - это переиначенное на английский лад выражение,
с помощью которого французские портовые грузчики просили коллег о
помощи. Какое слово мы заменили словом "ИКС"?

Ответ:
Mayday.

Зачет:
Мэйдэй; мэдэ.

Комментарий:
Речь идет о сигнале помощи "Mayday" [мэйдэй], который представляет собой
искаженное французское "m'aide" [мэд] - "помоги мне".

Источник:
Ю.И. Рылёв. 6000 изобретений XX и XXI веков, изменившие мир.
http://www.flibusta.is/b/377039/read

Автор:
Артем Матухно (Одесса)

Вопрос 12:
Рассказывают, что перед съемками "Терминатора" Арнольд Шварценеггер
специально подолгу проводил время на стрельбище, дабы научиться спускать
курок, не ДЕЛАЯ ЭТО. Как называется игра, в которой нельзя ДЕЛАТЬ ЭТО?

Ответ:
Гляделки.

Комментарий:
ДЕЛАТЬ ЭТО - моргать. Шварценеггер полагал, что роботы не моргают.

Источник:
   1. https://tjournal.ru/p/terminator-30-years
   2. http://ru.wiktionary.org/wiki/гляделки

Автор:
Артем Матухно (Одесса)

Тур:
Второй игровой день. 2 тур

Дата:
11-Nov-2016

Редактор:
Дмитрий Петров и Михаил Локшин (Санкт-Петербург)

Инфо:
Редакторы благодарят за тестирование вопросов и ценные замечания:
Алексея и Марию Трефиловых, Артема Корсуна, Владимира Салия, Кристину
Кораблину, Александра Мудрого, Сергея Терентьева, Сергея Лобачёва,
Ярослава Косарева, команду Павла Ершова и лично Павла Ершова, а также
команды "Кадис.ру", "Ноев ковчег" и "Постмодернистское название".

Вопрос 1:
В Крыму первый ОН появился благодаря Максимилиану Волошину. По мнению
Сергея Федина, по уму встречают только на НЕМ. Назовите ЕГО двумя
словами.

Ответ:
Нудистский пляж.

Комментарий:
Там по одежке не встречают. Волошину принадлежат труды "Блики. Нагота" и
"Блики. Маски. Нагота". И даже Ленин считал, что в движении нудистов
есть "здоровое пролетарское начало".

Источник:
   1. http://ru.wikipedia.org/wiki/Натуризм
   2. http://www.gramma.ru/RST/?id=3.810

Автор:
Михаил Локшин (Санкт-Петербург)

Вопрос 2:
Сахарную голову можно поджечь, если с ней СДЕЛАТЬ ЭТО. Впрочем, порой
ЭТО ДЕЛАЮТ не только с сахарной. Какие два слова на одну и ту же букву
мы заменили словами "СДЕЛАТЬ ЭТО"?

Ответ:
Посыпать пеплом.

Комментарий:
Пепел (в частности, табачный) содержит соли калия, в том числе поташ,
катализирующие горение сахарозы.

Источник:
   1. http://www.youtube.com/watch?v=bckN7iMhjmg
   2. http://p-i-f.livejournal.com/6982595.html

Автор:
Михаил Локшин (Санкт-Петербург)

Вопрос 3:
Виктор Гюго провел несколько лет в изгнании на скалистом острове Гернси.
Бодлер насмешливо писал, что терзаемый лишь уколами тщеславия Гюго любил
воображать себя... Кем?

Ответ:
Прометеем.

Комментарий:
Виктор Гюго, не принявший власть Наполеона III, был своего рода
бунтарем, как и Прометей. А Бодлер был весьма циничен и товарищей по
перу недолюбливал.

Источник:
А. Труайя. Бодлер. http://www.flibusta.is/b/291671/read

Автор:
Дмитрий Петров (Санкт-Петербург)

Вопрос 4:
Живший в XVI веке римский папа Сикст V любил грандиозные проекты. Так,
папа обещал оплатить ЕЕ создание, если ОНА доберется до цели. Для НЕЕ
была разработана специальная система сигналов. Назовите ЕЕ.

Ответ:
Непобедимая армада.

Зачет:
Испанская армада; Великая армада.

Комментарий:
Сикст V обещал оплатить создание армады после высадки испанцев в Англии.
В состав флота входили корабли нескольких государств. Специально для
Армады был создан интернациональный свод морских сигналов.

Источник:
   1. Р. Хьюз. Рим. История города: его культура, облик, люди. - М.:
АСТ: CORPUS, 2014. - С. 288.
   2. http://ru.wikipedia.org/wiki/Непобедимая_армада

Автор:
Дмитрий Петров (Санкт-Петербург)

Вопрос 5:
Внимание, в вопросе есть замена.
   Премьер-министр Турции на полном серьезе заявил: взгляды турецких
мусульман отличаются от взглядов боевиков ИГИЛ на целых ДВА ИКС.
Ответьте точно, что мы заменили словами "ДВА ИКС".

Ответ:
360 градусов.

Комментарий:
Ахмет Давутоглу сказал, что взгляды турецких мусульман отличаются от
взглядов террористов не на 180, а на целых 360 градусов. На всякий
случай упоминаем, что террористическая организация "Исламское
Государство" запрещена на территории РФ.

Источник:
http://www.middleeasteye.net/news/turkish-pm-mocked-over-360-degrees-difference-slip-190994676

Автор:
Дмитрий Петров (Санкт-Петербург)

Вопрос 6:
Пикап Toyota Hilux [тойОта хАйлакс] надежен и прост в ремонте. По словам
одного американского военнослужащего, Toyota Hilux стал автомобильным
эквивалентом ЕГО. Назовите ЕГО двумя словами.

Ответ:
Автомат Калашникова.

Комментарий:
Toyota Hilux из-за технических характеристик чаще всего используется как
импровизированная боевая машина в локальных конфликтах.

Источник:
   1. http://www.businessinsider.com/why-isis-uses-toyota-trucks-2015-10
   2. http://ru.wikipedia.org/wiki/Toyota_Hilux

Автор:
Дмитрий Петров (Санкт-Петербург)

Вопрос 7:
На фабрике ЕГО отца делали детали к самолетам "Мицубиси". Герой ЕГО
произведения говорит: "Самолет - это прекрасная мечта, а конструктор -
тот, кто ее воплощает". Назовите ЕГО.

Ответ:
[Хаяо] Миядзаки.

Комментарий:
Разнообразные летательные аппараты, в первую очередь самолеты, можно
увидеть во множестве его произведений: "Небесный замок Лапута", "Ветер
крепчает", "Навсикая из Долины Ветров", "Порко Россо" и других. Фразу из
вопроса произносит авиаконструктор из фильма "Ветер крепчает".

Источник:
   1. http://otium.su/xayao-miadzaki-velikij-fantazyor/
   2. http://ru.wikipedia.org/wiki/Миядзаки,_Хаяо
   3. Мультфильм "Ветер крепчает" (2013), реж. Хаяо Миядзаки, 12-я
минута. http://ru.wiki2.org/wiki/Ветер_крепчает?s=The%20Wind%20Rises

Автор:
Михаил Локшин (Санкт-Петербург)

Вопрос 8:
Разработанный Майком Томпсоном прибор для поиска сигнала Wi-Fi выглядит
как ОНА. Считается, что название деревни Аргури близ горы Арарат связано
с НЕЙ. Назовите ЕЕ одним словом.

Ответ:
Лоза.

Комментарий:
Дизайнер решил, что поиски сигнала Wi-Fi подобны поискам воды
лозоходцами. Как известно, на горе Арарат зреет красный виноград.

Источник:
   1. http://www.etoday.ru/2011/10/ustroystvo-dlya-poiska-wifi--.php
   2. http://ru.wikipedia.org/wiki/Лозоходство
   3. http://ru.wikipedia.org/wiki/Арарат
   4. http://ru.wikisource.org/wiki/ЭСБЕ/Аргури

Автор:
Дмитрий Петров (Санкт-Петербург)

Вопрос 9:
Механизмы работы иммунитета изучены недостаточно. Ученый Александр
ХаджИдис говорит, что иммунитет - это тонкая и пока еще ТАКАЯ ОНА.
Назовите ТАКУЮ ЕЕ.

Ответ:
Темная материя.

Комментарий:
Темная материя - гипотетическая форма материи, которая не испускает
электромагнитного излучения и напрямую не взаимодействует с ним. По
словам ученого, "иммунитет - это тонкая и пока еще темная материя".

Источник:
   1. http://doctorpiter.ru/articles/11136/
   2. http://ru.wikipedia.org/wiki/Тёмная_материя

Автор:
Михаил Локшин (Санкт-Петербург)

Вопрос 10:
Как-то раз сотрудники одного английского музея пришли на работу и
обнаружили, что ОНА XIX века до сих пор действует. Назовите ЕЕ.

Ответ:
Мышеловка.

Комментарий:
Несмотря на отсутствие приманки, музейная мышь забралась в старинную
мышеловку и не смогла оттуда выбраться.

Источник:
http://blogs.reading.ac.uk/merl/2016/02/03/155-year-old-mouse-trap-claims-its-latest-victim/

Автор:
Дмитрий Петров (Санкт-Петербург)

Вопрос 11:
Единственное слово, заимствованное английским языком из швейцарского
ретороманского, обозначает ЕЕ. Один из методов предсказания ИХ тоже
происходит из Швейцарии. Назовите ЕЕ.

Ответ:
Лавина.

Комментарий:
Английское слово "avalanche" [Эвэланш] - "лавина" - пришло из
швейцарского ретороманского через французский. Один из методов оценки
устойчивости снежно-ледового покрова был разработан в швейцарской армии.

Источник:
   1. Г. Доррен. Лингво. Языковой пейзаж Европы.
http://www.flibusta.is/b/446640/read
   2. http://ru.wikipedia.org/wiki/Лавина

Автор:
Дмитрий Петров (Санкт-Петербург)

Вопрос 12:
Древние китайцы верили, что ракУшки каУри приносят удачу, и придали
ЭТОМУ форму ракУшки каУри. Если вы решите приготовить ЭТО, лучше
использовать лазерный принтер - иначе будет трудно что-либо разобрать.
Назовите ЭТО тремя словами.

Ответ:
Печенье с предсказанием.

Зачет:
Печенье с предсказаниями.

Комментарий:
Выпекая из муки съедобное подобие каури, китайцы верили, что это
принесет им удачу. Если использовать струйный принтер, шариковую или
гелевую ручку, при высокой температуре текст на бумаге может смазаться.

Источник:
http://www.orientalica.com/kitayskoe-pechene-s-predskazaniyami-retseptyi-i-istoriya

Автор:
Михаил Локшин (Санкт-Петербург)

Тур:
Второй игровой день. 3 тур

Дата:
11-Nov-2016

Редактор:
Александр Мудрый (Черновцы)

Инфо:
Редактор благодарит за тестирование и ценные замечания: Ростислава
Гимчинского, Владимира Городецкого, Александра Зинченко, Ирину Зубкову,
Андрея Кокуленко, Николая Константинова, Артема Корсуна, Александра
Кудрявцева, Константина Науменко, Алексея и Марию Трефиловых, Игоря
Тюнькина, Ульяну Фабричнину, а также команду "Прометей" (Черновцы).

Вопрос 1:
Более четверти века в этом городе не функционировали полтора десятка
станций метрополитена. Назовите этот город.

Ответ:
Восточный Берлин.

Зачет:
Берлин.

Комментарий:
После сооружения Берлинской стены были закрыты семь станций на линии U6
и восемь станций на линии U8 в связи с тем, что эти линии шли из
западного сектора в западный через восточную часть. Было принято решение
не разрывать линии западного метрополитена, а только закрыть станции,
находящиеся в восточном секторе. Поезда через эти станции двигались не
останавливаясь.

Источник:
   1. http://ru.wikipedia.org/wiki/Берлинский_метрополитен
   2. http://forum.tr.ru/read.php?5,174129,page=all

Автор:
Александр Мудрый (Черновцы)

Вопрос 2:
На мемориальной доске Траяну Поповичу, который был мэром Черновцов в
середине прошлого века, есть число 19600. Сергей Воронцов назвал
Поповича "буковинским ИМ". Назовите ЕГО.

Ответ:
[Оскар] Шиндлер.

Комментарий:
В 1941 году Попович отказался выполнить приказ о депортации черновицких
евреев в лагеря Транснистрии. Мало того, он убедил вышестоящие власти в
своей правоте и добился изменения решения: 19600 черновицких евреев
остались в городе.

Источник:
   1. http://gazeta.zn.ua/SOCIETY/bukovinskiy_shindler.html
   2. https://pogliad.ua/news/chernivtsi/u-chernivcyah-vidkrili-memorialnu-doshku-trayanu-popovichu-280684

Автор:
Александр Мудрый (Черновцы)

Вопрос 3:
Большинство украинских телеканалов транслируются со спутников "Amos"
[Эймос], "Sirius" [сИриус] и "HotBird" [хотбёд]. Поэтому самый
распространенный на Украине тип спутниковых антенн получил сказочное
прозвище "он". В одном из слов предыдущего предложения мы пропустили
пять букв. Напишите это слово в исходном виде.

Ответ:
Горыныч.

Комментарий:
Чтобы поймать все три спутника, на тарелку устанавливают три конвертера,
которые в просторечии называют головками. Такая "трехголовая" антенна и
получила название в честь Змея из русских сказок.

Источник:
   1. http://www.satsis.info/forum (поиск по слову "горыныч")
   2. http://www.satsputnik.ru/vybrat-oborudovanie-dlya-prosmotra-besplatnogo-sputnikovogo-televideniya/

Автор:
Александр Мудрый (Черновцы)

Вопрос 4:
В первом тайме матча Евро-2016 Италия - Германия обе команды практически
не предпринимали атакующих действий. Денис Казанский назвал этот тайм
"ПРОПУСК ГарОнне". Заполните пропуск двумя или тремя словами.

Ответ:
Стояние на реке.

Зачет:
Стоянием на реке; стояние на; стоянием на.

Комментарий:
Матч проходил в Бордо, расположенном на реке Гаронне, и комментатор
сравнил такой футбол с известными событиями русской истории.

Источник:
Трансляция четвертьфинала Евро-2016 Италия - Германия на канале "Матч!
Футбол 1", эфир от 02.07.2016 г.

Автор:
Александр Мудрый (Черновцы)

Вопрос 5:
Одна из версий возникновения ЭТОГО - сбой в одном из отделов мозга. В
такой момент новая информация поступает в подкорковые ядра, отвечающие
за ее переработку, не напрямую, а через другую зону. Назовите ЭТО.

Ответ:
Дежавю.

Комментарий:
Из-за сбоя в парагиппокампальной извилине новая информация поступает в
подкорковые ядра через правую височную долю, отвечающую за средне- и
долговременную память. Поэтому человек воспринимает эту новую информацию
как уже виденную.

Источник:
http://pikabu.ru/story/prostoe_i_ponyatnoe_obyasnenie_pochemu_myi_ispyityivaem_dezhavyu_4484414

Автор:
Ростислав Гимчинский (Черновцы)

Вопрос 6:
После того как в болотах графства Кембриджшир было обнаружено деревянное
колесо, эту территорию прозвали местными ИМИ. Назовите ИХ одним словом.

Ответ:
Помпеи.

Комментарий:
В болотах при отсутствии доступа кислорода очень хорошо сохраняются
археологические находки. Место раскопок поселения близ Питерборо, в
котором нашли дубовое колесо возрастом 3000 лет, сами археологи назвали
самым богатым памятником бронзового века в Великобритании и сравнили с
легендарными Помпеями.

Источник:
https://www.theguardian.com/science/2016/feb/19/archaeologists-excavate-bronze-age-wheel-cambridgeshire

Автор:
Александр Мудрый (Черновцы)

Вопрос 7:
В одном сериале маленькая компания, разрабатывающая инновационную
технологию, узнаёт, что у нее появился серьезный конкурент. Один из
персонажей с грустью говорит, что их шансы опередить этого конкурента
ЧРЕЗВЫЧАЙНО МАЛЫ. Какое соотношение мы заменили словами "ЧРЕЗВЫЧАЙНО
МАЛЫ"?

Ответ:
Один к гуглу.

Зачет:
1:10^100.

Комментарий:
Конкурентом этой компании был "Google". Персонаж скаламбурил, понимая,
что шансы небольшого стартапа обогнать IT-гиганта очень невелики.

Источник:
Телесериал "Два с половиной человека", s11e16.

Автор:
Александр Мудрый (Черновцы)

Вопрос 8:
Внимание, ИКС в вопросе - замена.
   Получивший образование в Париже Тадеуш Костюшко устроился ИКСОМ,
чтобы быть поближе к любимой девушке. А заглавный персонаж известного
произведения стал ИКСОМ, чтобы быть поближе к своему врагу. Назовите
фамилию этого персонажа.

Ответ:
Дубровский.

Комментарий:
ИКС - гувернёр, учитель французского. Костюшко устроился гувернёром в
дом богатого помещика Сосновского, в дочь которого Людвику был влюблен.

Источник:
   1. Документальный фильм "Тадеуш Костюшко - польский генерал".
   2. http://ru.wikipedia.org/wiki/Дубровский_(роман)

Автор:
Александр Мудрый (Черновцы)

Вопрос 9:
Скоростной хоккеист Виталий Абрамов в прошлом сезоне ярко ворвался в
одну из юниорских лиг. Павел Климовицкий назвал Абрамова ИМ. Сам Абрамов
видел ЕГО, будучи в школе. Назовите ЕГО двумя словами.

Ответ:
Челябинский метеор.

Зачет:
Челябинский метеорит.

Комментарий:
Абрамов был признан лучшим новичком Главной юниорской хоккейной лиги
Квебека (QMJHL). Поскольку Виталий из Челябинска, то его не преминули
сравнить с метеоритом, пролетевшим над городом утром 15 февраля 2013
года. В это время 14-летний Абрамов, как и положено, находился в школе.

Источник:
   1. http://www.sport-express.ru/hockey/nhl/reviews/1014473/
   2. http://www.nhl.com/ice/ru/news.htm?id=886932

Автор:
Александр Мудрый (Черновцы)

Вопрос 10:
Наблюдение за одной экзопланетой поставило под сомнение эффективность
процесса циркуляризации. Заметка об этом называлась "ЧУДАКОВАТАЯ
экзопланета задает вопросы". Какое слово мы заменили словом
"ЧУДАКОВАТАЯ"?

Ответ:
Эксцентричная.

Зачет:
Эксцентрическая.

Комментарий:
Эта планета расположена в созвездии Большой Медведицы и представляет
собой газовый гигант размером с Юпитер. Она вращается вокруг своей
звезды по очень вытянутой, т.е. эксцентричной орбите. Процесс
циркуляризации состоит в "выравнивании" орбиты планеты, постепенном
приближении ее к круговой. Но в данном случае он происходит гораздо
медленнее, чем предсказывает теория.

Источник:
   1. http://www.nkj.ru/news/28463/
   2. http://dic.academic.ru/dic.nsf/dic_synonims/201591/

Автор:
Александр Мудрый (Черновцы)

Вопрос 11:
Рассказывая о заядлом рыбаке, Вашингтон Ирвинг пишет, что тот живет
жизнью настоящего ЕГО. ОНИ, вероятно, происходят от кистепёрых рыб.
Назовите ИХ двухкоренным словом.

Ответ:
Земноводные.

Комментарий:
Рыбак из рассказа Ирвинга в воде и около нее проводил не меньше времени,
чем на суше. Биологи полагают, что кистепёрые рыбы дали начало
земноводным и первыми из позвоночных вышли на сушу.

Источник:
   1. В. Ирвинг. Происшествие с черным рыбаком.
http://www.flibusta.is/b/70482/read
   2. http://ru.wikipedia.org/wiki/Происхождение_земноводных
   3. http://ru.wikipedia.org/wiki/Кистепёрые_рыбы

Автор:
Александр Мудрый (Черновцы)

Вопрос 12:
Назовите роман, первый перевод которого на русский язык выполнила в 1936
году Евгения Калашникова.

Ответ:
"Прощай, оружие!".

Комментарий:
Вот такое любопытное совпадение. Кстати, по мнению Корнея Чуковского,
этот перевод принадлежит к высшим достижениям советского переводческого
искусства.

Источник:
http://ru.wikipedia.org/wiki/Калашникова,_Евгения_Давыдовна

Автор:
Александр Мудрый (Черновцы)

Тур:
Третий игровой день. 1 тур

Дата:
09-Dec-2016

Редактор:
Антон Волосатов и Константин Сахаров (Ивантеевка)

Инфо:
Помощи в подготовке уделили свое ценное время: Никита Коровин (Манила),
Дмитрий Дягилев, Татьяна Левченко, Галина Пактовская, Серафим Шибанов
(все - Москва), Александр Усков и Олег Михеев (оба - Краснодар), Николай
Коврижных (Киров), Екатерина Сахарова (Ивантеевка).

Вопрос 1:
[Ведущему: отточие не озвучивать.]
   Заметка о чемпионате по какой игре вышла под заголовком "... Без
покемонов"?

Ответ:
Го.

Комментарий:
Мобильная игра "Pokémon Go" крайне популярна (или была таковой
несколько месяцев назад), но и поклонников интеллектуальной игры го в
мире насчитывают до 60 миллионов. Больше вопросов о покемонах и по
заголовкам в туре не будет!

Источник:
https://www.championat.com/other/article-252640-v-sankt-peterburge-prohodit-60-j-evropejskij-kongress-go.html

Автор:
Константин Сахаров (Ивантеевка)

Вопрос 2:
Однажды автор вопроса с друзьями играли в "Угадай кто", и одному из
игроков достался популярный персонаж американского мультсериала.
Воспроизведите первый вопрос, который задал о себе игрок.

Ответ:
"Я в данный момент умер?".

Зачет:
По упоминанию смерти или жизни.

Комментарий:
Кенни из "South Park" известен тем, что постоянно умирает и воскресает,
поэтому вопрос поставил партнеров в тупик.

Источник:
ЛОАВ.

Автор:
Константин Сахаров (Ивантеевка)

Вопрос 3:
Американская чемпионка мира в детстве нередко терпела оскорбления на
религиозной почве. Поэтому вид спорта был выбран такой, чтобы не
испытывать проблем с экипировкой. Назовите этот вид спорта.

Ответ:
Фехтование.

Комментарий:
Одежда девушек-мусульман не должна открывать руки и ноги. В фехтовании
тело и даже лицо спортсмена полностью закрыто униформой. Так что мама
разрешила ИбтихАдж МухаммАд заниматься спортом, увидев подходящие
костюмы фехтовальщиц. В Рио-де-Жанейро Мухаммад стала первой
американкой, которая выступала на Олимпиаде в хиджабе.

Источник:
   1. http://www.sports.ru/tribuna/blogs/innuendo/1020658.html
   2. http://ru.wikipedia.org/wiki/Мухаммад,_Ибтихадж

Автор:
Константин Сахаров (Ивантеевка)

Вопрос 4:
Белый крест на зеленом фоне - таким предлагали сделать флаг ЕЕ. Назовите
ЕЕ.

Ответ:
Гренландия.

Комментарий:
Гренландия - зависимая территория Дании, и собственный флаг ей
предлагали сделать по аналогии со всеми скандинавскими, но с учетом
"зеленого" названия территории. Вероятно, из-за стремления подчеркнуть
самостоятельность нынешний флаг скандинавского креста не содержит.

Источник:
http://ru.wikipedia.org/wiki/Флаг_Гренландии

Автор:
Константин Сахаров (Ивантеевка)

Вопрос 5:
Обсуждая фильм 2015 года, научный сотрудник Артем Климчук призвал ценить
знания, потому что они могут пригодиться в самый неожиданный момент. По
его мнению, от негативных коннотаций должно избавиться слово, означающее
специалиста. Напишите это слово.

Ответ:
Ботаник.

Комментарий:
В фильме "Марсианин" главный герой - по основной профессии ботаник -
сумел применить свои знания и выжить долгое время на Марсе. "Если
понятие "ботаник" воспринимать не как человека, который занимается
растениями, а как человека, который много чего знает и хорошо учился в
институте, то краткая мораль фильма - будьте ботаниками. Это вас
спасет".

Источник:
http://www.the-village.ru/village/weekend/oba/223751-the-martian-mfti

Автор:
Константин Сахаров (Ивантеевка)

Вопрос 6:
Комментируя итоги выборов осенью 2016 года, Эльвира Набиуллина заявила:
"Мы, конечно, все помним про песню". Из какого города группа,
исполнившая эту песню?

Ответ:
Из Владивостока.

Комментарий:
Выбор городов для размещения на новых рублевых купюрах завершился в
пользу Севастополя и Дальнего Востока. Решение о том, какой из городов
окажется на какой купюре, будет принимать совет директоров Центробанка
России (Набиуллина - его глава). Песня, о которой идет речь, -
"Владивосток 2000" группы "Мумий Тролль".

Источник:
http://www.vesti.ru/doc.html?id=2809499

Автор:
Константин Сахаров (Ивантеевка)

Вопрос 7:
В Эфиопии и Эритрее практикуется ОНА, включающая в себя такие стадии:
обжарка на углях, перемалывание в деревянной ступке, варка в котле,
перелив в другой сосуд и охлаждение, просеивание через фильтр из
конского волоса и наконец, разлив в кипящем состоянии. Назовите ЕЕ двумя
словами.

Ответ:
Кофейная церемония.

Комментарий:
Мы попытались дать описание, напоминающее более известную чайную
церемонию. Упомянутые африканские регионы известны своим кофе.

Источник:
http://www.intensocoffee.ru/articles.php?article=3_kofeinaja_ceremonia

Автор:
Антон Волосатов (Ивантеевка)

Вопрос 8:
На логотипе фирмы "Деловой звук" изображен несуществующий вид ЕГО -
скрипичный ключ. Назовите ЕГО двумя словами.

Ответ:
Галстучный узел.

Зачет:
Узел галстука; способ завязывания.

Комментарий:
Галстук - пожалуй, главнейший элемент делового стиля.

Источник:
http://www.business-sound.ru/

Автор:
Антон Волосатов (Ивантеевка)

Вопрос 9:
"Самый великий человек в мире - великий вождь Ким Ир Сен". Как
утверждает журнал "Максим", за эту фразу любого северокорейца отправили
бы в лагерь, так как ОНО не на месте. Напишите ЕГО.

Ответ:
Ким Ир Сен.

Зачет:
Имя [вождя].

Комментарий:
В КНДР имя Ким Ир Сена всегда должно стоять в начале предложения. Мы
нарушили это правило уже дважды. :-(

Источник:
http://www.maximonline.ru/longreads/get-smart/_article/north-korea/

Автор:
Илья Иванов (Москва)

Вопрос 10:
Когда ОНИ появились в Италии в начале прошлого века, за характерный
элемент ИХ стали называть "fumetti" [фумЕтти], что означает "дымок".
Ответьте одним словом, что такое ОНИ.

Ответ:
Комиксы.

Комментарий:
Итальянцам казалось, что спичбаблы в американских комиксах похожи на
облачка дыма, так что стали именовать так и сами комиксы.

Источник:
   1. Д.Е. Комм. Формулы страха. Введение в историю и теорию фильма
ужасов. http://www.flibusta.is/b/353326/read
   2. http://en.wikipedia.org/wiki/Italian_comics

Автор:
Руслан Хаиткулов (Москва)

Вопрос 11:
Искусствовед Эрвин ПанОфский пришел к выводу, что крылатый мальчик -
Эрот, а его связанные руки указывают на "вынужденное целомудрие".
Назовите заглавную героиню описанной картины.

Ответ:
Даная.

Комментарий:
Как известно, Зевс вступил в отношения с Данаей не какими-то привычными
методами, а в виде золотого дождя.

Источник:
http://ru.wikipedia.org/wiki/Даная_(картина_Рембрандта)

Автор:
Константин Сахаров (Ивантеевка)

Вопрос 12:
С 2016 года для НЕЕ официально введено ограничение в 45 секунд. В марте
1946 года Рэй МИлланд от НЕЕ отказался, ограничившись поклоном, что до
сих пор остается уникальным случаем. Назовите ЕЕ максимально точно.

Ответ:
Речь победителя на вручении премии "Оскар".

Зачет:
По словам "речь" и "Оскар".

Комментарий:
Чтобы слишком не затягивать церемонию и избежать инцидентов, выступления
победителей регламентировали. Вручение наград традиционно проводится на
границе зимы и весны. Несмотря на то что в карьере Милланда это была
первая и последняя статуэтка, он проявил необычайную скромность. А
награжден он был за лучшую мужскую роль в фильме "Потерянный уик-энд".
   z-checkdb: Ограничение в 45 секунд действует с 2010 года, см.
http://www.hollywoodreporter.com/lists/oscars-watch-10-longest-acceptance-867667/item/greer-garson-10-longest-acceptance-867635
(Евгений Рубашкин).

Источник:
   1. http://oscar2016.ru/news/45-seconds/
   2. http://ru.wikipedia.org/wiki/Рэй_Милланд

Автор:
Антон Волосатов (Ивантеевка)

Тур:
Третий игровой день. 2 тур

Дата:
09-Dec-2016

Редактор:
Александр Голиков и Людмила Полякова (Харьков)

Вопрос 1:
Героиня Набокова, опустив после сложного разговора телефонную трубку,
прислушалась и с облегчением услышала только ЕГО. Назовите ЕГО тремя
словами, начинающимися на одну и ту же букву.

Ответ:
Стук своего сердца.

Зачет:
Стучание своего сердца; стук/стучание собственного сердца.

Комментарий:
Она жила в большом доме и боялась, что этот разговор могли подслушать.
Рядом никого не оказалось. Это не "так судьба стучится в двери", конечно
же, но всё же.

Источник:
В.В. Набоков. Защита Лужина. http://www.flibusta.is/b/385732/read

Автор:
Людмила Полякова (Харьков)

Вопрос 2:
[Ведущему: внимательно прочитать конец первого предложения, чтобы
команды четко уяснили грамматическую структуру.]
   В одном фильме пилигрим сообщает, что первым по доблести был Ричард
Львиное Сердце, а ПРОПУСК. Уже в конце апреля 2016 года стало понятно,
что, несмотря на неверие букмекеров еще осенью, ПРОПУСК - и это как
минимум. Пропуски на слух неотличимы. Заполните любой из них двумя
словами.

Ответ:
Лестер второй.

Зачет:
"Лестер" - второй.

Комментарий:
Вторым был благородный граф Лестер.

Источник:
Х/ф "Баллада о доблестном рыцаре Айвенго" (1982), реж. Сергей Тарасов,
8-я минута.

Автор:
Александр Голиков (Харьков)

Вопрос 3:
[Ведущему: четко прочитать "в Шуне" и "в Кване".]
   В самом начале пьесы один из персонажей саркастично упоминает
случайность в Шуне и случайность в Кване. Впрочем, им все-таки больше
везет... Где?

Ответ:
В Сычуани.

Зачет:
В Сезуане.

Комментарий:
Боги в произведении Бертольта Брехта уже не верят, что им удастся найти
хотя бы одного доброго человека, но наконец-то нашелся хотя бы один - в
городе Сычуань.

Источник:
Б. Брехт. Добрый человек из Сычуани.
http://www.flibusta.is/b/469976/read

Автор:
Александр Голиков (Харьков)

Вопрос 4:
Когда брехтовские боги обсуждают водоноса, один из них обвиняет его в
мошенничестве, держа в руках кружку, из которой водонос их напоил водой.
Всё дело в НЕМ. Назовите ЕГО двумя словами, начинающимися на одну и ту
же букву.

Ответ:
Двойное дно.

Комментарий:
Водонос дал богам напиться воды из кружки с двойным дном, которое
символизирует нечестность и мошенничество.

Источник:
Б. Брехт. Добрый человек из Сычуани.
http://www.flibusta.is/b/469976/read

Автор:
Александр Голиков (Харьков)

Вопрос 5:
Внимание, в одном из слов вопроса мы пропустили две буквы.
   Вильфредо Парето - итальянский социолог-консерватор, который,
несмотря на свой аристократизм по происхождению, пережил семейную драму.
Преподаватель социологии, рассказывая об этом, заявил, что его жена,
Александра Бакунина, изменила ему с маркизом. Восстановите исходное
слово.

Ответ:
Марксизмом.

Комментарий:
Фамилия Александры Бакуниной явно указывает на ее марксистские и
анархистские контакты.

Источник:
   1. ЛНА на лекции.
   2. http://50.economicus.ru/index.php?ch=5&le=42&r=4&z=1
   3. http://ru.wikipedia.org/wiki/Парето,_Вильфредо

Автор:
Александр Голиков (Харьков)

Вопрос 6:
В мае 1949 года ОН передал свой список из 38 журналистов и писателей
спецслужбам. В этом списке были такие характеристики, как "бесчестный
карьерист", "сентиментальный симпатизант", "еврейский",
"антибританский", "хорошо нажился в СССР", "очень антибелый" и тому
подобное. Назовите ЕГО.

Ответ:
[Эрик] Блэр.

Зачет:
[Джордж] Оруэлл.

Комментарий:
Да, марксист по убеждениям Эрик Блэр работал на Большого Брата.

Источник:
http://gefter.ru/archive/15452

Автор:
Александр Голиков (Харьков)

Вопрос 7:
Согласно одному несерьезному источнику, в результате забастовки цыган
тысячи горожан остались ПРОПУСК. По мнению профессора Криса Синхи,
представители племени амондава живут ПРОПУСК, поэтому могут говорить
только об одном событии. Какие два слова на одну и ту же букву мы дважды
пропустили?

Ответ:
Без будущего.

Комментарий:
Вряд ли это племя живет в мире Оруэлла, но будущего времени в его языке
действительно нет.

Источник:
   1. http://www.anekdot.ru/id/256514/
   2. http://www.bbc.com/russian/international/2011/05/110520_amondawa_language_lacks_time.shtml

Автор:
Людмила Полякова (Харьков)

Вопрос 8:
   <раздатка>
   leaned
   </раздатка>
   Седьмой эпизод "Звездных войн" многие обвиняют во вторичности. В
неологизме, который газета "Вашингтон Пост" применяет по отношению к
одной из планет в этом фильме, мы убрали четыре буквы. Восстановите
исходное слово.

Ответ:
Alderaaned.

Комментарий:
Планета была уничтожена, как и Алдераан, причем как способ уничтожения,
так и кинокартинка дают основания подозревать создателей фильма во
вторичности.

Источник:
https://www.washingtonpost.com/news/monkey-cage/wp/2016/01/06/here-are-three-ways-that-star-wars-is-a-neoconservative-universe/

Автор:
Людмила Полякова (Харьков)

Вопрос 9:
Как утверждают журналисты, город Вольфсбург подхватывает воспаление
легких, когда корпорация "Фольксваген" ДЕЛАЕТ ЭТО. Желая показать, что
он опознал фрагменты плагиата, Джоакино Россини на концерте неоднократно
ДЕЛАЛ ЭТО. Что делал?

Ответ:
Снимал шляпу.

Комментарий:
От "Фольксвагена" сильно зависит благосостояние Вольфсбурга. А Россини
"здоровался" со "встреченными" им во время концерта коллегами.

Источник:
   1. "Футбол", 2015, N 66 (1352). - С. 21.
   2. А.И. Муха. Музыканты смеются. http://www.flibusta.is/b/273591/read

Автор:
Александр Голиков (Харьков)

Вопрос 10:
В книге "Алиса и Алисия" положительный персонаж Алиса Селезнёва
приезжает на Суматру за одним экземпляром, путешествует во времени и
знакомится с диктатором и тираном Алисией I. Какое слово мы пропустили в
предыдущем предложении?

Ответ:
Бабочки.

Комментарий:
Как можно понять из имени, Алисия - негативное развитие Алисы.
Получается своеобразный эффект бабочки.

Источник:
К. Булычёв. Алиса и Алисия. http://www.flibusta.is/b/108182/read

Автор:
Александр Голиков (Харьков)

Вопрос 11:
Известный персонаж книги Бориса Акунина при виде Тенерифе произносит,
как это ни странно для русскоязычного читателя, короткое слово. Назовите
это двусложное слово.

Ответ:
Яма.

Комментарий:
Тенерифе - это остров со знаменитым вулканом Тейде. Персонаж, Масахиро
Сибата, - японец, слуга Фандорина, и для него это "яма" (сравните с
"Фудзи-яма", "Тате-яма").

Источник:
   1. Б. Акунин. Планета Вода. http://www.flibusta.is/b/401666/read
   2. http://ru.wikipedia.org/wiki/Тенерифе
   3. http://ru.wikipedia.org/wiki/Фудзияма
   4. http://ru.wikipedia.org/wiki/Татеяма_(гора)

Автор:
Людмила Полякова (Харьков)

Вопрос 12:
В постмодернистской пародии на пьесу "Ричард III" Эдвард расстраивается,
что его после смерти не слушают, и упоминает о планах переезда. Ответьте
абсолютно точно: куда именно?

Ответ:
Эльсинор.

Зачет:
Замок Эльсинор.

Комментарий:
И, как и подобает в постмодернистском тексте, Эдвард становится тенью -
видимо, тенью отца Гамлета.

Источник:
http://the-mockturtle.livejournal.com/770332.html

Автор:
Людмила Полякова (Харьков)

Тур:
Третий игровой день. 3 тур

Дата:
09-Dec-2016

Редактор:
Олег Михеев и Александр Усков (Краснодар)

Инфо:
Редакторы благодарят за тестирование вопросов и ценные советы: Андрея
Баландина и Андрея Гречишникова (оба - Москва), Максима Карачуна
(Краснодар), Маргариту Кирюшину и Инессу Кличманову (обе - Москва), Илью
Кукушкина и Дениса Лагутина (оба - Краснодар), Наталью Мойсик
(Ростов-на-Дону), Марию Подрядчикову (Волгоград), Павла Солюкова
(Краснодар), Дмитрия Тарарыкова и Артема Шинкевича (оба - Москва).

Вопрос 1:
По легенде, художник ЗЕвксис не смог найти достаточно красивую девушку,
для того чтобы изобразить Елену Прекрасную. Мэтью ГАмперт сравнивает
ЗЕвксиса с заглавным героем произведения XIX века. Назовите этого героя.

Ответ:
[Виктор] Франкенштейн.

Комментарий:
Легенда гласит, что ЗЕвксис отобрал для позирования пятерых девушек и
позаимствовал лучшие черты каждой из них для портрета Елены. В романе
Мэри Шелли Франкенштейн создал своего монстра, используя различные
фрагменты тел.

Источник:
   1. Matthew Gumpert. Grafting Helen: The Abduction of the Classical
Past.
https://books.google.ru/books?id=5TM1hIAfX-EC&pg=PA258#v=onepage&q&f=false
   2. http://ru.wikipedia.org/wiki/Виктор_Франкенштейн

Автор:
Олег Михеев (Краснодар)

Вопрос 2:
Желающие поучаствовать в проводимом в Сахаре марафоне должны оплатить
вступительный взнос. В русское специализированное название того, для
чего, помимо прочего, предназначен этот взнос, входит число. Напишите
это число.

Ответ:
200.

Комментарий:
Марафон является достаточно опасным, так что этот взнос предназначен на
случай смерти участника и репатриации его тела на родину. В вооруженных
силах тела погибших принято называть "грузом 200".

Источник:
   1. https://www.si.com/vault/2002/04/29/8103066/hot--cold-at-the-marathon-des-sables-billed-as-the-worlds-toughest-footrace-even-the-stoutest-of-competitors-couldnt-avoid-getting-saharan-sand-kicked-and-blown-in-their-faces
   2. http://ru.wikipedia.org/wiki/Груз_200

Автор:
Олег Михеев (Краснодар)

Вопрос 3:
Рассказывают, что каждый день с февраля 1858 года на протяжении
четырнадцати лет Бобби можно было встретить на кладбище ГрейфрАйерс в
Эдинбурге. В статье журнала "National Geographic" [нэшнл джиогрЭфик]
Бобби называют предшественником... Кого?

Ответ:
Хатико.

Комментарий:
Скайтерьер Бобби стал знаменит тем, что охранял могилу хозяина, а затем
был похоронен недалеко от ворот кладбища. Вскоре в Эдинбурге был открыт
памятник Бобби. Более известна похожая история про японского пса Хатико.

Источник:
http://www.nat-geo.ru/fact/38617-predannyy-pes-bobbi/

Автор:
Александр Усков (Краснодар)

Вопрос 4:
По одной из версий, альбом известной группы был назван в честь команды
для устройств Macintosh [макинтОш]. Сейчас схожая по структуре команда
используется в ИКСЕ. На вышеупомянутом альбоме ИКС тоже есть. Назовите
ИКС одним словом.

Ответ:
Android.

Зачет:
Андроид.

Комментарий:
Фраза "OK Computer", ставшая названием альбома группы "Radiohead"
[рэйдиохЭд], была командой для голосового распознавания в Macintosh. В
операционной системе Android для той же цели используется команда "OK
Google". Композиция "Paranoid Android" [паранОид андрОид] входит в
вышеупомянутый альбом.

Источник:
http://ru.wikipedia.org/wiki/OK_Computer

Автор:
Александр Усков (Краснодар)

Вопрос 5:
По официальной версии, в Краснодаре весной этого года ОНО не состоялось
по соображениям безопасности. Кроме того, представитель епархии упомянул
негативные ассоциации с древним кельтским обычаем. Назовите ЕГО.

Ответ:
Сожжение [чучела] Масленицы.

Зачет:
Сжигание [чучела] Масленицы.

Комментарий:
Как один из ближайших аналогов русского обычая, представители епархии
упомянули обряд жертвоприношения друидов, в процессе которого жертва
заключается в специальную клетку, называемую плетеным человеком, и
подвергается сожжению.

Источник:
   1. https://www.yuga.ru/news/392580/
   2. http://ru.wikipedia.org/wiki/Плетёный_человек

Автор:
Александр Усков (Краснодар)

Вопрос 6:
В этом вопросе словом "ИКС" мы заменили другое слово.
   Одна из статей американского закона устанавливает, в каких случаях
кража считается совершённой в крупном размере. Например, если сумма
украденного превышает 950 долларов или если украден ИКС. С этой статьей
связывают происхождение фразы из трех слов, зачастую сокращаемой до
аббревиатуры. Назовите эту аббревиатуру.

Ответ:
GTA.

Зачет:
ГТА.

Комментарий:
Выражение, которым названа популярная компьютерная игра, появилось еще в
середине века. Согласно законам Калифорнии, одним из условий, когда
кража считается крупной, является угон автомобиля, вне зависимости от
его цены. Так в уголовный жаргон вошло выражение "Grand Theft Auto"
[гранд зэфт Ото]. Этим, кстати, объясняется и ненатуральный для
английского языка порядок слов, ведь более правильным и привычным было
бы "Auto Grand Theft".

Источник:
http://leginfo.legislature.ca.gov/faces/codes_displaySection.xhtml?lawCode=PEN&sectionNum=487

Автор:
Олег Михеев (Краснодар)

Вопрос 7:
Согласно не вполне серьезному совету, британцам не стоит покупать
красные автомобили. Дело в том, что заскучавшие полицейские иногда
соревнуются в так называемой "дорожной" разновидности... Ответьте словом
английского происхождения: разновидности чего?

Ответ:
Снукер.

Комментарий:
Снукер - это разновидность бильярда, в который играют 15 красными шарами
и 6 шарами других цветов, причем забиваться красные и цветные шары
должны поочередно. Популярная в Англии городская легенда гласит, что
полицейские иногда играют в снукер машинами, т.е. останавливают машины в
соответствии с их цветом и правилами снукера, поэтому водители красных
автомобилей имеют наибольший шанс быть остановленными.

Источник:
   1. http://www.exposedpolice.com/uk-traffic-police-playing-motorway-snooker/
   2. http://ru.wikipedia.org/wiki/Снукер

Автор:
Олег Михеев (Краснодар)

Вопрос 8:
По одной из версий, причиной тяжелых последствий происшествия в Альпах
могла стать установленная на НЕМ камера. На НЕМ можно было увидеть семь
звезд. Назовите ЕГО двумя словами, начинающимися на одну и ту же букву.

Ответ:
Шлем Шумахера.

Комментарий:
В 2013 году Михаэль Шумахер во время катания на лыжах упал и получил
серьезную травму головы. Существует версия, что причиной этого могла
стать камера GoPro [гОу про], установленная на шлеме и разбившая его при
ударе. Согласно распространенному заблуждению, семь звезд на гоночном
шлеме Шумахера олицетворяют семь чемпионских титулов "Формулы-1", но на
самом деле такой дизайн шлема был у него практически с начала карьеры.

Источник:
   1. http://www.telegraph.co.uk/sport/motorsport/formulaone/michael-schumacher/10640839/Michael-Schumacher-skiing-crash-did-helmet-camera-cause-head-injuries.html
   2. http://www.alamy.com/stock-photo-michael-schumacher-a-racing-helmet-of-the-seven-time-formula-1-world-31707308.html

Автор:
Олег Михеев (Краснодар)

Вопрос 9:
Согласно одному шуточному изображению, ученому удалось создать видимость
того, что он всё еще работает над НЕЙ. Однако очевидно, что работа уже
завершена. Назовите ЕЕ словом греческого происхождения.

Ответ:
Голограмма.

Зачет:
Голография.

Комментарий:
Ученый создал голограмму самого себя, которая изображает его за работой.

Источник:
http://www.ifunny.com/pictures/greg-was-still-hard-work-hologram-technology/

Автор:
Александр Усков (Краснодар)

Вопрос 10:
В начале XX века Старый Том помогал китобоям в охоте, взамен получая
часть добычи. В одной статье это сотрудничество охарактеризовано
термином, впервые употребленным в современном значении в 1940 году.
Назовите этот термин словом с удвоенной согласной.

Ответ:
Коллаборационизм.

Комментарий:
Старый Том был вожаком стаи косаток, сопровождавших усатых китов в
бухту, где охотились австралийские китобои. Иногда косатки даже хватали
зубами веревку гарпуна и помогали в буксировке; взамен китобои оставляли
им языки убитых китов. Коллаборационизм - это осознанное и добровольное
сотрудничество с врагом. Впервые так стали называть сотрудничавшее с
немцами правительство ВишИ во Франции.

Источник:
   1. http://www.jewishhistoryaustralia.net/Nulla_Nulla_Story/A_5_Killers_in_Eden.htm
   2. http://en.wikipedia.org/wiki/Collaborationism#Etymology

Автор:
Олег Михеев (Краснодар)

Вопрос 11:
Журналист и критик Алекс Шепард известен скептическими высказываниями.
Одно из них привело к тому, что в октябре этого года Шепард опубликовал
фотографию пластинки Боба Дилана, на которой лежат два металлических
предмета. Назовите эти предметы.

Ответ:
Вилка, нож.

Зачет:
В любом порядке.

Комментарий:
Боб Дилан регулярно появлялся в списке фаворитов на получение
Нобелевской премии по литературе. Несмотря на это, Алекс Шепард
неоднократно заявлял, что не верит в победу Дилана, а в 2015 году даже
пообещал съесть свой экземпляр пластинки "Blood on the Tracks" [блад он
зэ трэкс], если это всё же случится.

Источник:
   1. https://twitter.com/alex_shephard/status/786530758571724800/
   2. https://newrepublic.com/article/123058/who-will-win-nobel-prize-literature

Автор:
Александр Усков (Краснодар)

Вопрос 12:
Проведя раскопки в одном американском городе на улице КонтИ, ученые
обнаружили, в частности, много румян и бутылок из-под ликера. Это
позволило предположить, что заведение, упоминаемое в известном
произведении, действительно существовало. Назовите это произведение.

Ответ:
"The House of the Rising Sun" [чтецу: зэ хАус оф зэ рАйзин сан].

Зачет:
Дом восходящего солнца.

Комментарий:
"Дом восходящего солнца" - народная американская песня. Обычно под ним
понимают то ли тюрьму, то ли бордель середины XIX века в Новом Орлеане.
Существовало ли это заведение, до сих пор остается открытым вопросом. По
словам археолога Шэннон Доуди, проведшей раскопки на предполагаемом
месте, известном из газет того времени, обстановка выглядела похожей на
бордель.

Источник:
   1. http://en.wikipedia.org/wiki/The_House_of_the_Rising_Sun
   2. http://www.angelpig.com/house_history.html

Автор:
Олег Михеев (Краснодар)

Тур:
Финал (Минск). 1 тур

Дата:
25-Mar-2017

Редактор:
Ольга Кузьма и Андрей Кузьма (Санкт-Петербург)

Инфо:
Редакторы тура благодарят за помощь и ценные замечания Алексея Рабина,
Максима Веслополова, Евгения и Аллу Муштай (все - Санкт-Петербург) и
Дмитрия Свинтицкого (Могилев).

Вопрос 1:
Из первоначального списка претендентов на звание "музыкальный символ
чемпионата мира - 2018" сразу были вычеркнуты все духовые инструменты.
Причиной такого решения стали ОНИ. Назовите ИХ иностранным словом.

Ответ:
Вувузелы.

Комментарий:
Вувузелы - музыкальный символ чемпионата мира по футболу 2010 года -
многие до сих пор вспоминают с ужасом. "Вувузела" в переводе с
зулусского - "делать шум". Музыкальным символом ЧМ-2018 стали ложки.

Источник:
   1. http://radiozenit.ru/news_full/uid/621
   2. http://ru.wikipedia.org/wiki/Вувузела

Автор:
Ольга Кузьма, Андрей Кузьма (Санкт-Петербург)

Вопрос 2:
Уильяма Фридмана, пытавшегося предупредить ВМФ США о налёте на
Пёрл-Харбор, называют американским ИМ. ОН родился в 1895 году в
Бакинской губернии. Назовите ЕГО.

Ответ:
[Рихард] Зорге.

Источник:
Станислав Зигуненко. 100 великих загадок истории флота. - М.: Вече,
2012. - С. 223.

Автор:
Ольга Кузьма, Андрей Кузьма (Санкт-Петербург)

Вопрос 3:
Адель Алексеева пишет, что на рисунке Пушкина, в отличие от работ многих
художников, ОН не смешной, а скорее благородный, породистый. Назовите
ЕГО двумя словами, не используя кавычки.

Ответ:
Нос Гоголя.

Источник:
Адель Алексеева. Как влюблялись, творили и шалили наши классики. - М.:
Вече, 2016. - С. 53.

Автор:
Ольга Кузьма, Андрей Кузьма (Санкт-Петербург)

Вопрос 4:
Известный журналист полагал, что будет похоронен рядом с человеком, чей
девиз - "Христианство, торговля и цивилизация", но настоятель
Вестминстерского аббатства воспротивился этому. Назовите фамилию этого
журналиста.

Ответ:
Стэнли.

Комментарий:
Слово "полагал" - подсказка. Генри Мортон Стэнли нашел пропавшего в
Африке Давида Ливингстона и приветствовал его словами "Доктор
Ливингстон, полагаю?".

Источник:
   1. Великие путешествия. Герои, покорившие планету. - СПб.:
"Пресс-курьер", N 14 за 2016 год. - С. 104.
   2. http://ru.wikipedia.org/wiki/Ливингстон,_Давид

Автор:
Ольга Кузьма, Андрей Кузьма (Санкт-Петербург)

Вопрос 5:
Болельщики "Манчестер Юнайтед" дали лидерам своего клуба Полю Погба,
Златану Ибрагимовичу и Уэйну Руни очень короткое прозвище, намекающее на
фатальность действий данных футболистов для соперников. Напишите это
прозвище по-английски.

Ответ:
RIP.

Комментарий:
Прозвище образовано по первым буквам фамилий игроков. RIP - сокращение
от "rest in peace" [рест ин пис]. "Покойся с миром" - традиционная
надпись на надгробьях в англоязычных странах.

Источник:
https://www.championat.com/football/news-2543880-trojke-igrokov-mju-runi-ibragimovich-pogba-dali-shutlivoe-prozvische-r-i-p.html

Автор:
Ольга Кузьма, Андрей Кузьма (Санкт-Петербург)

Вопрос 6:
На Каспии издавна было принято расстилать в прибрежной полосе тряпки и
старые ковры, прижимая их камнями. Таким образом местные жители ДЕЛАЛИ
ЭТО для домашнего использования. Россия входят в тройку лидеров среди
стран, ДЕЛАЮЩИХ ЭТО. Какие слова мы заменили словами "ДЕЛАТЬ ЭТО"?

Ответ:
Добывать нефть.

Источник:
Станислав Зигуненко. 100 великих загадок истории флота. - М.: Вече,
2012. - С. 157.

Автор:
Ольга Кузьма, Андрей Кузьма (Санкт-Петербург)

Вопрос 7:
В вопросе есть замена.
   Известный гурман Александр Дюма писал: "Я немного поохотился на
берегах Каспия, где в таком же изобилии водятся дикие гуси, утки,
пеликаны, как на Сене - собаки". Восстановите замененное нами слово.

Ответ:
Лягушки.

Источник:
Великие путешествия. Герои, покорившие планету. - СПб.: Пресс-курьер, N
14 за 2016 год. - С. 141.

Автор:
Ольга Кузьма, Андрей Кузьма (Санкт-Петербург)

Вопрос 8:
Майкл Крайтон пишет, что в период правления бельгийской колониальной
администрации случаи каннибализма стали более редкими, а к началу 1960-х
годов в Конго даже появилось несколько ИХ. Одно из НИХ фигурирует в
заглавии романа 1983 года. Назовите автора этого романа.

Ответ:
[Стивен] Кинг.

Комментарий:
ОНИ - кладбища. Роман - "Кладбище домашних животных".

Источник:
   1. М. Крайтон. Конго. http://flibusta.is/b/135768/read
   2. http://ru.wikipedia.org/wiki/Кладбище_домашних_животных_(роман)

Автор:
Ольга Кузьма, Андрей Кузьма (Санкт-Петербург)

Вопрос 9:
Для примата галаго из семейства лориобразных слух не менее важен, чем
зрение. За внешний вид галаго часто называют именем персонажа,
появившегося около полувека назад. Назовите этого персонажа.

Ответ:
Чебурашка.

Комментарий:
У Галаго не только глаза, но и уши велики.

Источник:
Дмитрий Бердышев. Самые необычные животные. - М.: ЭНАС-КНИГА, 2016. - С.
97, 101.

Автор:
Ольга Кузьма, Андрей Кузьма (Санкт-Петербург)

Вопрос 10:
Однажды Роберт Вуд растопил несколько килограммов свинца и вылил их в
небольшое отверстие в земле. После того как свинец застыл, Вуд, потратив
на раскопки несколько часов, извлек его, получив своеобразный
скульптурный портрет ЕЕ. Назовите ЕЕ.

Ответ:
Молния.

Комментарий:
Разветвленная отливка ушла в землю на глубину более трех метров.

Источник:
З. Столбовский. Великие тайны и загадки мира. Опасности и угрозы. - М.:
Мартин, 2005. - С. 156.

Автор:
Ольга Кузьма, Андрей Кузьма (Санкт-Петербург)

Вопрос 11:
26 апреля 1945 года Николай Масалов спас ребенка. Через несколько дней в
его полк приехал человек, поговоривший с Николаем и сделавший несколько
набросков его внешности. Назовите этого человека.

Ответ:
[Евгений] Вучетич.

Комментарий:
Впоследствии Вучетич создал знаменитый памятник, установленный в
Трептов-парке.

Источник:
Вячеслав Бондаренко. 100 великих подвигов России. - М.: Вече, 2014. - С.
306.

Автор:
Ольга Кузьма, Андрей Кузьма (Санкт-Петербург)

Вопрос 12:
В 1966 году отряд советских подводных лодок совершил кругосветное
путешествие, не всплывая на поверхность. За время плавания советские
моряки за ненадобностью ни разу не совершили некое действие. Шесть лет
назад в Белоруссии это действие сочли ненужным. Назовите ЕГО.

Ответ:
Перевод часов.

Источник:
   1. Станислав Зигуненко. 100 великих загадок истории флота. - М.:
Вече, 2012. - С. 246.
   2. http://ru.wikipedia.org/wiki/Время_в_Белоруссии

Автор:
Ольга Кузьма, Андрей Кузьма (Санкт-Петербург)

Тур:
Финал (Минск). 2 тур

Дата:
25-Mar-2017

Редактор:
Александр Кудрявцев (Николаев)

Инфо:
Редактор благодарит за тестирование вопросов Тараса Вахрива, Дмитрия
Великова, Анастасию Гончарову, Екатерину Дубровскую, Андрея Кокуленко,
Юлию Лунёву, Максима Мерзлякова, Дениса Обуха, Дмитрия Овчарука, Алексея
Рабина, Аркадия Руха и Владимира Шлапака.

Вопрос 1:
(pic: 20160954.jpg)
   На раздаточном материале - рентгеновский ИКС, которым чаще других
пользовались девушки, имевшие ближневосточные или средиземноморские
корни. Девушек привлекало то, что эффект держался всю жизнь, а процедура
была абсолютно безболезненной. Назовите ИКС.

Ответ:
Эпилятор.

Зачет:
Депилятор.

Комментарий:
У девушек с ближневосточной или средиземноморской кровью нередко на лице
росли усики. С помощью рентгеновского облучения рост волос
останавливался. Правда, спустя какое-то время начинали проявляться
нежелательные последствия.

Источник:
http://www.cosmeticsandskin.com/cdc/xray.php

Автор:
Александр Кудрявцев (Николаев)

Вопрос 2:
После того как французский король Филипп I обозвал Вильгельма
Завоевателя бабой, последний некоторое время ограничивался только
спиртными напитками, надеясь исправить ситуацию. Какое слово с удвоенной
согласной мы пропустили в тексте вопроса?

Ответ:
Беременной.

Комментарий:
Вильгельм сильно растолстел и, после того как Филипп обозвал его
беременной бабой, решил похудеть, прибегнув к диете, которая состояла
исключительно из спиртных напитков.

Источник:
http://www.history.com/news/history-lists/10-things-you-may-not-know-about-william-the-conqueror

Автор:
Александр Кудрявцев (Николаев)

Вопрос 3:
Слово "ИКС" является заменой.
   Мать легендарного налетчика Джесси Джеймса тоже была не промах. После
смерти сына она организовала прибыльный бизнес, скупая старые ИКСЫ и
потом продавая их существенно дороже. Английский этимологический словарь
сообщает, что слово "ИКС" происходит от глагола со значением "свистеть".
Назовите ИКС.

Ответ:
Пистолет.

Комментарий:
Мать Джесси Джеймса обманывала покупателей, говоря, что все продаваемые
ею пистолеты принадлежали ее сыну. Английский этимологический словарь
указывает на происхождение слова "пистолет" от чешского
"pi&scaron;t'ala" [пистАла], которое, в свою очередь, происходит от
глагола "pisteti" [пистЕти] ("свистеть"). Однако если вы подумали, что
слово происходит от английского глагола "to whistle" [ту висл], - ничего
страшного.

Источник:
   1. http://en.wikipedia.org/wiki/Zerelda_James
   2. http://www.etymonline.com/index.php?term=pistol&allowed_in_frame=0

Автор:
Александр Кудрявцев (Николаев)

Вопрос 4:
Когда наивные телезрители стали интересоваться, как вырастить дерево у
себя дома, компания BBC [би-би-си] ответила, что саженец следует
посадить в жестянку с НИМ и надеяться на лучшее. Назовите ЕГО двумя
словами, которые начинаются на соседние буквы.

Ответ:
Томатный соус.

Комментарий:
В 1957 году в телепрограмме BBC "Панорама" в качестве первоапрельской
шутки был показан сюжет о необычайно богатом урожае макаронных деревьев
в Швейцарии.

Источник:
http://en.wikipedia.org/wiki/Spaghetti-tree_hoax

Автор:
Александр Кудрявцев (Николаев)

Вопрос 5:
   <раздатка>
   низм
   </раздатка>
   Существует вероучение, признаЮщее и уважающее все религии, в том
числе буддизм. Какие буквы мы пропустили в названии этого вероучения?

Ответ:
ом.

Комментарий:
Ом - широко употребляемая в буддизме мантра. "Omni" [Омни] - латинский
префикс, означающий "все" или "каждый".

Источник:
   1. http://en.wikipedia.org/wiki/Omnism
   2. http://en.wikipedia.org/wiki/Omni
   3. http://ru.wikipedia.org/wiki/Ом_(мантра)

Автор:
Александр Кудрявцев (Николаев)

Вопрос 6:
(pic: 20160955.jpg)
   Мадрид расположен в предгорьях массива СьЕрра-де-ГвадаррАма. В статье
об изображенной на раздаточном материале мадридской статуе сообщается и
ее высота над уровнем моря. Укажите эту высоту.

Ответ:
666 метров.

Комментарий:
Это статуя Падшего Ангела в мадридском парке БуЭн-РетИро. Согласно
христианскому вероучению, падшие ангелы - это ангелы, взбунтовавшиеся
против Бога и прОклятые им за это. Первым и наиболее известным падшим
ангелом является Сатана.

Источник:
http://en.wikipedia.org/wiki/Fuente_del_%C3%81ngel_Ca%C3%ADdo

Автор:
Александр Кудрявцев (Николаев)

Вопрос 7:
(pic: 20160956.jpg)
   Через несколько часов после сооружения изображенная на снимке
пирамида была сожжена. Назовите двумя словами причину сожжения.

Ответ:
Сухой закон.

Зачет:
Запрет алкоголя.

Комментарий:
Пирамида сооружена из бочек с конфискованным алкоголем, которые через
несколько минут будут сожжены.

Источник:
http://www.kulturologia.ru/blogs/031115/27002/

Автор:
Александр Кудрявцев (Николаев)

Вопрос 8:
В прошлом веке в США группы добровольцев во избежание трагедий совершали
поисковые рейды по свалкам и повреждали ИХ, отрывая двери или выламывая
замки. Назовите ИХ.

Ответ:
Холодильники.

Комментарий:
В старых холодильниках замок не позволял открыть дверцу изнутри, и дети
нередко погибали, забравшись внутрь во время игр. Чтобы этого случайно
не произошло на свалке, добровольцы повреждали выброшенные холодильники.

Источник:
http://en.wikipedia.org/wiki/Refrigerator_death

Автор:
Александр Кудрявцев (Николаев)

Вопрос 9:
В XVII веке английские моряки стали кое-что делать на своем корабле в
знак скорби по погибшим товарищам, объясняя, что на время траура
освобождают место для невидимого ИКСА смерти. Так, по одной из версий,
появилась распространенная современная традиция. Назовите ИКС.

Ответ:
Флаг.

Комментарий:
Речь идет о традиции во время траура приспускать флаг.

Источник:
https://www.washingtonpost.com/news/morning-mix/wp/2015/07/22/a-sign-of-death-not-division-the-bloody-history-behind-lowering-flags-to-half-staff/

Автор:
Александр Кудрявцев (Николаев)

Вопрос 10:
В 30-е годы в СССР профессия летчика считалась верхом престижа. Поэтому
в одной телепрограмме утверждалось, что в указанное время мужчина в
лётной форме во всех смыслах являлся ИМ. Назовите ЕГО двукоренным
словом.

Ответ:
Небожитель.

Источник:
Хроники московского быта. Сын Кремля.
http://www.tvc.ru/channel/brand/id/37/show/episodes/episode_id/45326/

Автор:
Александр Кудрявцев (Николаев)

Вопрос 11:
Самолет, на котором летает американский президент, называют "Air Force
One" [эйр форс ван]. Прозвище самолета, на котором летает родившийся в
2012 году Американский Фараон, двумя буквами отличается от названия
президентского самолета. Напишите это прозвище.

Ответ:
"Air Horse One" [чтецу: эйр хорс ван].

Комментарий:
Американский Фараон - кличка знаменитого скакуна. К местам, где проходят
скачки, его доставляют специальным самолетом.

Источник:
   1. http://www.eonline.com/news/663024/kentucky-derby-champion-american-pharoah-flies-on-a-plane-called-air-horse-one-yes-seriously
   2. http://en.wikipedia.org/wiki/American_Pharoah
   3. http://en.wikipedia.org/wiki/Air_Force_One

Автор:
Александр Кудрявцев (Николаев)

Вопрос 12:
(pic: 20160957.jpg)
   Поскольку фамилией ИКСА являлось старинное английское слово,
переводчик Владимир МЕдек использовал старочешское слово с тем же
значением, имевшее к тому же определенную фонетическую общность.
Назовите ИКСА.

Ответ:
[Альбус] Дамблдор.

Комментарий:
На раздаточном материале слово "шмель" написано на старочешском и
современном чешском языках. Фамилия Альбуса Дамблдора происходит от
староанглийского слова со значением "шмель"; сравните со словом
"bumblebee" [бАмблби]. Имя "Альбус" значит "белый", поэтому шмель на
раздаточном материале - белый.

Источник:
http://en.wikipedia.org/wiki/Harry_Potter_in_translation

Автор:
Александр Кудрявцев (Николаев)

Тур:
Финал (Минск). 3 тур

Дата:
25-Mar-2017

Редактор:
Алексей Полевой (Гомель) и Денис Рыбачук (Брест)

Инфо:
Редакторы благодарят за помощь в работе над пакетом Максима Мерзлякова
(Воронеж), Сергея Терентьева и Бориса Моносова (оба - Санкт-Петербург),
Алексея и Марию Трефиловых (Калуга), Александра Кудрявцева (Николаев),
Николая Слюняева (Нижний Новгород), Наиля Фарукшина (Навои - Москва),
Дмитрия Капитанюка (Брест), Максима Новика (Гомель), Александра Огнева
(Краков), Арсэна Атнагулова (Уфа), Тараса Вахрива (Тернополь), Игоря
Тюнькина (Москва), Андрея Кокуленко (Омск).

Вопрос 1:
В русском переводе одного мультфильма гигант, насмехаясь над
противником, нараспев произносит обидное слово и повторяет его последний
слог. Напишите это слово.

Ответ:
Блоха.

Зачет:
Блоха-ха-ха и т.п.

Комментарий:
По сравнению с гигантом противник был очень мал. Переводчика этого
эпизода, видимо, вдохновила (пауза и ... если есть возможность, то
поставить музыкальный фрагмент отсюда (только звук, видео не надо) -
www.youtube.com/watch?v=LXELFcq5qJE с 46 секунды) песня Мефистофеля
композитора Мусоргского в исполнении Федора Шаляпина.

Источник:
   1. Мультфильм "Геркулес" (студия "Дисней", 1997).
   2. "Блоха" (Песня Мефистофеля в погребке Ауэрбаха), композитор М.П.
Мусоргский, слова И.В. Гёте, исполняет Ф.И. Шаляпин.
http://www.youtube.com/watch?v=LXELFcq5qJE

Автор:
Денис Рыбачук (Брест)

Вопрос 2:
В 1893 году иностранная фирма установила в одном из зданий российского
города лифт. Лев КЕкушев усовершенствовал его конструкцию и получил
премию. Что, согласно шутке современников, сделал с лифтом Кекушев?

Ответ:
Подковал [его].

Комментарий:
Похожая история произошла и с героем Лескова, подковавшим английскую
блоху. Фирма, правда, была не английская, а немецкая, город - не Тула, а
Москва, но современники шутили, что премия "за блоху, кою Лёва
подковал".

Источник:
http://moscow-i-ya.livejournal.com/368515.html

Автор:
Алексей Полевой (Гомель)

Вопрос 3:
В прошлые века при поиске залежей руды разведчики внимательно смотрели
под ноги во время поездки. Увиденная ОНА, по мнению Тимофея БажЕнова,
могла привести к открытию месторождения, рядом с которым запылают
плавильные печи. Назовите ЕЕ.

Ответ:
Искра.

Комментарий:
Разведчики смотрели под ноги лошади. Подкованное копыто в местах,
богатых металлическими рудами, могло высечь искру. Если месторождение
было богатым, то вскоре из искры могло возгореться пламя плавильных
печей. Стихотворение Пушкина с соответствующей строкой, кстати,
называется "Во глубине сибирских руд".

Источник:
Рейтинг Тимофея Баженова. Человек для опытов. Хозяйка медной горы.
https://russia.tv/video/show/brand_id/3879/episode_id/170455/

Автор:
Денис Рыбачук (Брест)

Вопрос 4:
Герои одного романа плохо разбираются в дореволюционных реалиях. Они
предполагают, что жившие в подвалах рабочие, чтобы выйти на солнечный
свет, должны были купить ИКС. Какие два слова мы заменили ИКСОМ?

Ответ:
Желтый билет.

Зачет:
Заменительный билет.

Комментарий:
Они думают, что билет желтый, так как по нему можно было выйти на
солнце. На самом деле желтый билет давал возможность легально заниматься
проституцией. Редакторы не исключают возможности того, что в зале есть
игроки, которые знали, что официальным названием "желтого билета" было
"заменительный билет", и ответили на вопрос, воспользовавшись
словом-подсказкой "заменили".

Источник:
   1. О.Д. Форш. Сумасшедший корабль. http://flibusta.is/b/268668/read
   2. http://ru.wikipedia.org/wiki/Жёлтый_билет

Автор:
Денис Рыбачук (Брест)

Вопрос 5:
В биографии Ильи Ильфа отмечается, что в первые дни после переезда в
Москву журналист искал для себя газету, отдавая предпочтение
широкоформатным. Это объяснялось отсутствием ЕГО. Какое произведение с
"НЕГО" начинается?

Ответ:
Мойдодыр.

Комментарий:
В то время Ильф был беден. Возможно, разыскивая газету пошире, чтобы
удобнее было укрываться, он повторял строчки недавно написанной
Чуковским сказки:
   Одеяло убежало,
   Улетела простыня,
   И подушка, как лягушка,
   Ускакала от меня.

Источник:
   1. Александра Ильф "Дом, милый дом".
http://tfile.co/forum/viewtopic.php?t=840279
   2. http://www.stihi-rus.ru/1/chukovskiy/14.htm

Автор:
Денис Рыбачук (Брест)

Вопрос 6:
Строительство началось во время Великой Депрессии, дало работу и помогло
выжить многим людям. Объект оказался очень дорогим - сооружение обошлось
в 35 миллионов долларов. Назовите этот объект.

Ответ:
Мост "Золотые ворота".

Комментарий:
На тот момент "Золотые ворота" оказались действительно "золотыми", за
период строительства начальная смета выросла вдвое. К сожалению, сейчас,
находясь в депрессии, многие используют мост совсем не по назначению.

Источник:
   1. http://gelio.livejournal.com/223154.html
   2. http://vse-chudesa.ru/chudesa-sveta-noveyshee-vremya/most-zolotie-vorota-v-san-francisko.html
   3. http://ru.wikipedia.org/wiki/Золотые_Ворота_(мост)

Автор:
Алексей Полевой (Гомель)

Вопрос 7:
В армии древних греков ИКСЫ одними из первых вступали в битву. Вскоре
после Берлинской олимпиады копия "ИКСА" оказалась в Германии. Какое
слово мы заменили ИКСОМ?

Ответ:
Дискобол.

Комментарий:
Диск использовался и как дальнее метательное оружие. Адольф Гитлер
считал статую "Дискобола" образцом для подражания и купил в Италии одну
из двух сохранившихся копий статуи Мирона. Кстати, статуя появляется в
начале фильма Лени Рифеншталь "Олимпия".

Источник:
   1. Документальный фильм "Происхождение Олимпийских игр" (Olympia -
The Origins of Games) (2016).
http://tfile.co/forum/viewtopic.php?t=888142
   2. http://www.bbc.com/russian/society/2015/04/150408_vert_cul_hitlers_idea_of_the_perfect_body

Автор:
Денис Рыбачук (Брест)

Вопрос 8:
Сотрудник американского журнала вспоминал, как в один прекрасный день
увидел ИКСА, который схватил АЛЬФУ быстро, как вспышка, и что обратил
внимание на контраст между ИКСОМ и АЛЬФОЙ. Назовите ИКСА и АЛЬФУ
словами, начинающимися на одну и ту же букву.

Ответ:
Матрос, медсестра.

Зачет:
Моряк, медсестра.

Комментарий:
(pic: 20160958.jpg)
   В один прекрасный день, когда американцы узнали об окончании войны,
они вышли на улицы, и начались гуляния. Фотограф Альфред Эйзенштадт
вспоминал, что матрос носился по улице быстро и целовал многих женщин,
но кадр с медсестрой получился особенно эффектным из-за контраста между
темной формой матроса и белым халатом медсестры. Дело было в Нью-Йорке,
а Нью-Йорк, как известно, - город контрастов.

Источник:
http://www.kp.by/daily/26401/3277924/

Автор:
Алексей Полевой (Гомель)

Вопрос 9:
Персонажа современного романа выгоняют из ТАКОГО клуба, после того как
тот использует банку с краской в качестве импровизированной ЕЕ. Назовите
ТАКУЮ ЕЕ.

Ответ:
Пейнтбольная граната.

Комментарий:
Персонаж использовал банку с краской как гранату, но в пейнтбольном
клубе его находчивость не оценили.

Источник:
Т. Пратчетт. Джонни и бомба. http://flibusta.is/b/486418/read

Автор:
Денис Рыбачук (Брест)

Вопрос 10:
КакАпо по кличке СирОкко нередко участвует в международных
природоохранных конференциях и выставках. По мнению создателей одного
документального фильма, СирОкко, в отличие от других какАпо, нельзя
назвать... Каким?

Ответ:
Нелетающим.

Комментарий:
Новозеландские какАпо - нелетающие птицы, как и киви, но Сирокко
является послом своего вида и символом охраны природы в Новой Зеландии,
поэтому часто летает на различные международные конференции, правда, на
самолетах.

Источник:
Документальный цикл "Дикая природа Новой Зеландии" (Wild New Zealand)
(2016), 3-я серия. http://tfile.co/forum/viewtopic.php?t=885320

Автор:
Денис Рыбачук (Брест)

Вопрос 11:
Яркие личинки жука мАйки, стремясь попасть в ИКС, собираются на кончиках
травинок. Назовите ИКС.

Ответ:
Улей.

Комментарий:
Нелетающие, но яркие личинки образуют подобие цветка и таким образом
приманивают пчел. Впоследствии они цепляются за пчелу и отпускают только
тогда, когда та прилетит в улей. Последствия для пчелиной семьи бывают
трагичными, так как личинки начинают поедать всё, что найдут в улье.

Источник:
Документальный цикл "Яд. Достижение эволюции" (2015), 3-я серия.
http://tfile.co/forum/viewtopic.php?t=878522

Автор:
Денис Рыбачук (Брест)

Вопрос 12:
В одном романе детей сначала приучают к взаимовыручке, а уже потом к
самостоятельности. Поэтому до определенного возраста глаза у детей
сзади, а потом - спереди. Какое слово в вопросе мы заменили?

Ответ:
Пуговицы.

Комментарий:
Когда все пуговицы на одежде сзади, то одеться ты можешь только с
чьей-либо помощью, а когда спереди, то уже и самостоятельно.

Источник:
Л. Лоури. Дающий. http://flibusta.is/b/236576/read

Автор:
Алексей Полевой (Гомель)

Тур:
Финал (Минск). 4 тур

Дата:
25-Mar-2017

Редактор:
Серафим Шибанов (Москва), при участии Александра Карясова (Самара)

Инфо:
Сделать пакет лучше помогали: Анастасия Белова, Глеб Крутинин, Никита
Поздняков (все - Пущино), Виктория Бочкарёва, Сергей Кухарев, Дарья
Макушова, Юрий Мотькин, Регина Шарипова (все - Самара), Александр
Кудрявцев (Николаев), а также команды "Приматы" и "Слон потрогает тебя"
(обе - Самара).

Вопрос 1:
Компания "Alphabet" [Альфабет] перенесла свой сайт на новый домен,
который позиционируется как альтернатива .com [дот ком]. По мнению
автора вопроса, схожим образом в своем логотипе подчеркивает
разнообразие предоставляемых услуг и другая компания. Назовите эту
другую компанию.

Ответ:
"Amazon".

Комментарий:
Новый сайт "Alphabet" [Альфабет] находится по адресу abc.xyz [эй-би-си
дот икс-вай-зет], т.е., можно сказать, от A [эй] до Z [зет]. В логотипе
"AmaZon" [амазОн] от буквы A [эй] к букве Z [зет] идет стрелочка.

Источник:
   1. https://abc.xyz/
   2. http://en.wikipedia.org/wiki/Alphabet_Inc.
   3. https://www.amazon.co.uk/

Автор:
Илья Иванов (Путилково)

Вопрос 2:
"Гамбург" - единственный футбольный клуб, который играл во всех сезонах
немецкой Бундеслиги. За это клуб получил прозвище "ОН". Назовите фильм
1993 года, в котором можно увидеть более десятка ИХ.

Ответ:
"Парк Юрского периода".

Комментарий:
Клуб прозвали Динозавром (иногда встречается вариант "Динозавр
Бундеслиги"). В фильме "Парк Юрского периода" много разных динозавров.

Источник:
   1. http://en.wikipedia.org/wiki/Hamburger_SV
   2. http://en.wikipedia.org/wiki/Jurassic_Park_(film)

Автор:
Серафим Шибанов (Москва)

Вопрос 3:
В песне группы "25/17" [двадцать пять семнадцать] отмечается, что
реальность и так достаточно сурова, и говорится, что бояться нужно не
ВТОРЫХ, а ПЕРВЫХ. Кто создал произведение, в заглавии которого
фигурируют ПЕРВЫЕ и ВТОРЫЕ?

Ответ:
[Константин] Симонов.

Комментарий:
Строчка из песни звучит так: "Все боятся мертвых, а нужно живых".

Источник:
   1. http://rusrap.org.ru/mp3/2517/text/Cherep_i_kosti.htm
   2. http://ru.wikipedia.org/wiki/Живые_и_мёртвые_(роман)

Автор:
Серафим Шибанов (Москва)

Вопрос 4:
Надеемся, этот вопрос будет вам приятен.
   В конце XIX века Лондон часто называли выражением, которое буквально
переводится как "Большой ОН". Согласно поговорке, ОН тесно связан... С
чем?

Ответ:
С огнем.

Комментарий:
Лондон известен своим смогом, поэтому получил прозвище Big Smoke [биг
смОук], что буквально переводится как "Большой дым". Согласно поговорке,
дыма без огня не бывает. В начале вопроса есть отсылка к цитате из "Горя
от ума" "И дым отечества нам сладок и приятен".

Источник:
   1. Дж.Э. Гарднер. Возвращение Мориарти.
http://flibusta.is/b/264025/read
   2. http://slovarick.ru/374/

Автор:
Серафим Шибанов (Москва)

Вопрос 5:
В своей книге Крис Тёрни поступил с погубившим древнюю сосну ученым так,
как должны были поступить с НИМ. В каком городе ОН жил?

Ответ:
Эфес.

Комментарий:
Крис Тёрни решил, что не стоит упоминать имя ученого, уничтожившего
столь ценный объект. Тот, кого древние греки запретили вспоминать, жил в
Эфесе и сжег местный храм Артемиды.

Источник:
   1. К. Тёрни. Кости, скалы и звезды. Наука о том, когда что произошло.
http://flibusta.is/b/320384/read
   2. http://en.wikipedia.org/wiki/Herostratus

Автор:
Игорь Тюнькин (Москва)

Вопрос 6:
Футболист МАриан ЧИшовски окончил карьеру в 2014 году. Незадолго до
завершения карьеры спортсмен принял участие в ЭТОМ с подачи хоккеиста
ВАцлава ПлЕтки. Назовите ЭТО тремя английскими словами.

Ответ:
Ice Bucket Challenge [чтецу: айс бАкет чЕллендж].

Зачет:
Айс бакет челлендж.

Комментарий:
Чишовски принял участие в флешмобе под названием "Ice Bucket Challenge"
[айс бакет чЕллендж], который должен был привлечь внимание к боковому
амиотрофическому склерозу. К сожалению, через пару месяцев после этого
Мариан узнал, что сам болен этой болезнью.

Источник:
http://www.sports.ru/tribuna/blogs/superratings/1182036.html

Автор:
Александр Карясов (Самара)

Вопрос 7:
[Ведущему: выделить голосом слово "распалась".]
   В 1918 году ЕМУ предложили должность в университете города Черновцы.
Пока ОН думал, ехать или не ехать, Австро-Венгрия распалась, и ОН
отказался. Назовите ЕГО.

Ответ:
[Эрвин] Шрёдингер.

Комментарий:
А если бы Австро-Венгрия не распалась, Шрёдингер принял бы это
предложение. В знаменитом мысленном эксперименте кот жив, если атом
радиоактивного вещества не распался, и мертв, если распался.

Источник:
http://ru.wikipedia.org/wiki/Шрёдингер,_Эрвин

Автор:
Александр Карясов (Самара)

Вопрос 8:
   <раздатка>
   В памятник Захер-Мазоху во Львове вмонтирована ОНА, через которую
можно разглядывать сменяющиеся эротические картинки. ONA изображена на
памятнике в европейской столице вместе... С кем?
   </раздатка>
   Вопрос перед вами. Время!

Ответ:
С Ромулом и Ремом.

Комментарий:
В первом случае имеется в виду лупа как прибор для улучшения зрения, а
во втором - Lupa capitolina - "Капитолийская волчица".

Источник:
   1. http://www.travel.ru/news/2008/03/24/121343.html
   2. http://it.wikipedia.org/wiki/Lupa_capitolina

Автор:
Александр Карясов (Самара)

Вопрос 9:
Советский ученый Владимир СперАнтов рассказывал, как его смелый знакомый
заявил, что устал от происходящего, и ушел к Сергею. Назовите фамилию
Сергея.

Ответ:
Прокофьев.

Комментарий:
Сергей Генкин, как и многие советские люди, присутствовал на похоронах
Сталина. Однако ему надоела толпа людей, и поэтому он захотел пойти на
похороны Прокофьева, который также умер 5 марта 1953 года.

Источник:
http://050353.ru/2016/03/04/sperantov/

Автор:
Александр Карясов, в редакции Юрия Мотькина (оба - Самара)

Вопрос 10:
ЕЕ дочерей зовут ОктАвия и СеверИна. В одном интервью ОНА призналась,
что в детстве была на домашнем обучении. Кто ОНА?

Ответ:
[Валерия Гай] Германика.

Комментарий:
Создательница сериала "Школа" сама в школу толком и не ходила.

Источник:
   1. http://ru.wikipedia.org/wiki/Германика,_Валерия_Гай_Александровна
   2. http://www.razgovorchiki.ru/arkhiv/germanika.htm

Автор:
Серафим Шибанов (Москва)

Вопрос 11:
В статье о НЕМ упоминается старовенгерский алфавит, придуманный
пастухами. Напишите слово, которое мы обозначили как ОН, при помощи
НЕГО.

Ответ:
   Бустро
   нодеф

Зачет:
Часть слова "бустрофедон" должна быть написана в одном направлении,
часть - в другом.

Комментарий:
Бустрофедон - способ письма, при котором чередуются строки, идущие слева
направо и справа налево.

Источник:
http://en.wikipedia.org/wiki/Boustrophedon

Автор:
Александр Карясов (Самара)

Вопрос 12:
Сериал "Во все тяжкие" повествует об Уолтере Уайте, которому
диагностируют неоперабельный рак. Вскоре после окончания сериала в
издании "Albuquerque Journal" [альбукЕрке джОрнал] появился ОН. Назовите
ЕГО точно.

Ответ:
Некролог [Уолтера] Уайта.

Комментарий:
Фанаты не поленились, и не важно, что Уолтер Уайт был всего лишь
персонаж.

Источник:
http://variety.com/2013/tv/news/breaking-bad-walter-white-obit-1200694265/

Автор:
Серафим Шибанов (Москва)

Тур:
Финал (Минск). 5 тур

Дата:
25-Mar-2017

Редактор:
Мишель Матвеев (Санкт-Петербург)

Инфо:
Редактор благодарит тестеров: Антон Тахтаров, Яна Азриэль, Дмитрий
Овчарук, Наталья Орлова, Сергей Терентьев, Александр Коробейников,
Владимир Бройда, Александр Камаев, Юлия Фукельман, Сергей Лобачёв,
Владимир Городецкий, Ирина Зубкова, Антон Волосатов, Валерий Юдачёв,
Илья Чадаев, Андрей Данченко, Алексей Акименко, Глеб Олейник, Андрей
Кокуленко, Ваган Калайджян, Садиг Гамидов, Артем Рожков.

Вопрос 1:
Один человек был изуродован своими врагами, но в Петербурге нашелся
искусный специалист, который ему помог. Исследователь Крашенинников
усматривает в этом основу сюжета произведения. Какого?

Ответ:
"Нос".

Комментарий:
Упомянутому человеку враги отрезали нос, а петербургский мастер
изготовил для него протез. По мнению Крашенинникова, искусственный нос
мог навеять Гоголю фантазию о самостоятельных прогулках носа коллежского
асессора Ковалева.

Источник:
http://magazines.russ.ru/voplit/2001/5/kpas.html

Автор:
Мишель Матвеев (Санкт-Петербург)

Вопрос 2:
ОНИ сильно ограничивают свободу действий и поэтому, согласно Википедии,
стали символом женского целомудрия и мужской власти. Назовите процедуру,
результатом которой ОНИ становились.

Ответ:
Бинтование ног.

Зачет:
Забинтовывание ног; перебинтовывание ног; бинтовка ног.

Комментарий:
Женщина с ногами, изуродованными бинтованием, была ограничена в
возможности передвигаться самостоятельно, она была вынуждена сидеть дома
и не могла пойти куда-либо без сопровождения слуг.

Источник:
http://ru.wikipedia.org/wiki/Бинтование_ног

Автор:
Мишель Матвеев (Санкт-Петербург)

Вопрос 3:
Право посетить непосредственно сам ИКС получает очень небольшое
количество людей. В частности, один раз это могут сделать нобелевские
лауреаты. Назовите ИКС двумя словами.

Ответ:
Корабль "Ваза".

Зачет:
Корабль "Васа".

Комментарий:
Музей корабля "Ваза" в Стокгольме ежедневно посещает множество людей, но
экспозиция расположена вокруг корабля, а сам корабль берегут и
посетителей на него пускают лишь в исключительных случаях. В частности,
раз в году после вручения Нобелевской премии лауреаты получают право на
посещение.

Источник:
   1. Рассказ экскурсовода в музее "Ваза".
   2. http://cr2.livejournal.com/84353.html

Автор:
Мишель Матвеев (Санкт-Петербург)

Вопрос 4:
К герою Акунина, лежащему в реанимации после автокатастрофы, приходит
посетитель. Белье героя он называет словом женского рода. Каким?

Ответ:
Рубашка.

Зачет:
Сорочка.

Комментарий:
Идиома "родиться в рубашке" означает "быть везучим, счастливо избегать
беды". По мнению посетителя, белье героя, выжившего в автокатастрофе, -
это и есть та рубашка, в которой он родился.

Источник:
Б. Акунин. Фантастика. http://flibusta.is/b/182057/read

Автор:
Мишель Матвеев (Санкт-Петербург)

Вопрос 5:
На знаменитом мероприятии охрана может проверить любого посетителя,
выходящего из палатки. Так предотвращаются тысячи ИХ. Назовите ИХ тремя
словами.

Ответ:
Кражи пивных кружек.

Зачет:
Похищения пивных кружек; кражи/похищения пивных бокалов.

Комментарий:
Мероприятие - Октоберфест. Кружки крадут не только ради кружек, но еще и
просто потому, что это стало популярным развлечением. Приходится
принимать меры.

Источник:
http://ru.wikipedia.org/wiki/Октоберфест

Автор:
Мишель Матвеев (Санкт-Петербург)

Вопрос 6:
Микробиолог Блэкмор выяснил, что некоторые бактерии могут двигаться
необычным образом благодаря содержащимся в их организме кристаллическим
цепочкам железа. Аналогом чего являются эти цепочки?

Ответ:
Компаса.

Комментарий:
Эти бактерии могут двигаться строго на север или на юг, поскольку
упомянутые цепочки вытягиваются вдоль линий магнитного поля Земли.

Источник:
   1. С.И. Венецкий. В мире металлов. http://flibusta.is/b/224963/read
   2. http://ru.wikipedia.org/wiki/Магнетосома

Автор:
Мишель Матвеев (Санкт-Петербург)

Вопрос 7:
Сергей Бубка пошутил, что, когда в одном государстве происходит некое
событие, стоит такой рев, что его слышно в соседних государствах.
Назовите это событие.

Ответ:
Гран-при Монако.

Зачет:
Гонки "Формула-1" и т.п. по словам "Формула-1".

Комментарий:
Имеется в виду рев моторов. Монако - совсем маленький анклав внутри
Франции, а расстояние до итальянской границы составляет всего десяток
километров. Сергей Бубка, как и ряд других спортсменов, по окончании
карьеры поселился в Монако.

Источник:
https://esquire.ru/wil/bubka

Автор:
Мишель Матвеев (Санкт-Петербург)

Вопрос 8:
Дуплет.
   1. Персонаж одного романа называл ЕЕ Венерой. Назовите ЕЕ двумя
словами.
   2. Персонажу классического произведения в критический момент
показалось, что ОНА - это аристократка. Назовите ЕЕ двумя словами.

Ответ:
   1. Дама червей.
   2. Дама пик.

Комментарий:
   1. Венера - богиня любви, сердце - символ любви и символ червовой
масти.
   2. В "Пиковой даме" Германну в момент проигрыша померещилось, что
дама пик была старухой-графиней.

Источник:
   1. Б. Акунин. Азазель. http://flibusta.is/b/228606/read
   2. А.С. Пушкин. Пиковая дама.
http://www.rvb.ru/pushkin/01text/06prose/01prose/0866.htm

Автор:
Мишель Матвеев (Санкт-Петербург)

Вопрос 9:
Гравитон - это гипотетическая частица, переносящая гравитационное
взаимодействие. Часто задаются вопросом, почему ОН не оказывается
непреодолимым для гравитонов. Назовите ЕГО.

Ответ:
Горизонт событий [черной дыры].

Комментарий:
С одной стороны, черная дыра гравитационно взаимодействует с окружающим
миром. С другой стороны, согласно общей теории относительности, ничто, в
числе гравитон, не может выбраться из-за ее горизонта событий. Есть
несколько возможных способов разрешения этого противоречия - например,
гравитация может распространяться виртуальными гравитонами, рождающимися
вблизи горизонта событий. Окончательный ответ, видимо, зависит от того,
какой будет теория, в рамках которой будет описан гравитон.

Источник:
   1. https://www.quora.com/Can-gravitons-escape-the-event-horizon-of-a-black-hole-If-not-how-can-we-detect-the-black-holes-gravity
   2. http://physics.stackexchange.com/questions/107185/are-gravitons-bound-by-the-event-horizon
   3. http://curious.astro.cornell.edu/physics/89-the-universe/black-holes-and-quasars/theoretical-questions/451-how-do-gravitons-escape-black-holes-to-tell-the-universe-about-their-gravity-advanced
   4. https://www.google.ru/?gws_rd=ssl#q=graviton+event+horizon

Автор:
Мишель Матвеев (Санкт-Петербург)

Вопрос 10:
Согласно несерьезному мнению, это произведение следует признать
экстремистским за описание убийства главы государства при помощи дрона.
Назовите это произведение.

Ответ:
"Сказка о золотом петушке".

Комментарий:
Золотой петушок, убивший царя Додона, - вполне себе дрон. А домик Элли,
если что, не был управляемым, да и Гингема - не глава государства.

Источник:
http://www.anekdot.ru/id/822764/

Автор:
Мишель Матвеев (Санкт-Петербург)

Вопрос 11:
Одна из версий объясняет происхождение известного спортивного термина
так: он образован от французского слова "l'&oelig;uf" [лёф], означающего
яйцо, а яйцо похоже на... Что?

Ответ:
Ноль.

Комментарий:
В теннисе ноль очков почему-то называют словом "love" [лав]. Причины и
пытается объяснить приведенная версия.

Источник:
http://en.wikipedia.org/wiki/Tennis_scoring_system

Автор:
Мишель Матвеев (Санкт-Петербург)

Вопрос 12:
В рассказе об известном произведении говорится, что в Карлсбаде некто
пошутил: "Видите, как действуют на организм карлсбадские воды!".
Назовите это произведение.

Ответ:
"Прощальная симфония".

Комментарий:
По версии шутника, музыканты один за другим уходили со сцены по личным
мотивам. Кстати, исполнялась не сама "Прощальная симфония" Гайдна, а
произведение Рихарда Штрауса, использовавшее ту же идею постепенного
ухода музыкантов.

Источник:
http://www.blagaya.ru/put/articles/muzhumor/

Автор:
Мишель Матвеев (Санкт-Петербург)


FreeBSD-CVSweb <freebsd-cvsweb@FreeBSD.org>